Está en la página 1de 35

1.

Cul de los siguientes pacientes tiene una indi


cacin ms clara de tratamiento quirrgico de
enfermedad por reflujo?:

1.

1.

3.

2.

3.

4.

5.

2.

4.
5.

5.

5.

Se trata de un divertculo por traccin.


Suele diagnosticarse en pacientes jvenes.
El tratamiento incluye la miotoma del msculo
cricofaringeo.
La pirosis es su sntoma principal.
Se localiza siempre en la cara anterior de la
hipofaringe.

La gastroscopia es una prueba diagnstica fun


damental para el estudio del paciente con
hemorragia digestiva, y la urgencia en la reali
zacin de la misma depender de la magnitud
de la hemorragia digestiva. Podra sealar
cul de las siguientes situaciones NO expresa
una mayor magnitud y por tanto gravedad de la
hemorragia?:
1.
2.
3.
4.

5.

4.

4.

Cul de las siguientes afirmaciones es correcta


en relacin al divertculo faringoesofgico o
divertculo de Zenker?:
1.
2.
3.

3.

Paciente de 80 aos con hernia hiatal de mediano tamao y pirosis frecuente que responde
bien a 20 mg/da de omeprazol.
Paciente de 56 aos con molestias epigstricas
tipo flatulencia y pirosis intermitente que responde slo ligeramente al tratamiento con
omeprazol.
Paciente de 27 aos con sndrome depresivo y
molestias retroestemales que no alivian en absoluto con el tratamiento con inhibidores de la
bomba de protones.
Paciente de 58 aos con pirosis diaria diurna y
nocturna de ms de 10 aos de evolucin y
que permanece asintomtico en los perodos que
es tratado con dosis de omeprazol de 40
mg/da o superiores y cuyos sntomas recidivan
inmediatamente al reducir esta dosis.
Paciente de 66 aos de edad con esofagitis
erosiva y antecedentes de infarto de miocardio
hace 1 ao con insuficiencia cardiaca residual.

2.

Hipotensin y taquicardia.
Disminucin del hematocrito y hemoglobina,
Pacientes que requieren transfusin para mantener la estabilidad hemodinmica.
Lavado gstrico por sonda naso-gstrica con
abundante sangre roja, que no se aclara tras
lavados repetidos con abundante volumen.
Repetidos y frecuentes episodios de hematemesis con sangre roja y melenas.

Cul de las siguientes pautas es ms probable


que sea efectiva en la erradicacin de
Helicobcer Pylori despus del fracaso de un
primer tratamiento con Omeprazol (20
mg/12h), Amoxicina (lg/12h) y Claritromicina
(500 mg/12h) durante siete das?:

Un paciente de 46 aos de edad acude a urgen


cias por vmitos en posos de caf y melena. El
hematocrito de entrada es del 33%, su presin
arterial es de 110/70 mmHg y su frecuencia
cardiaca de 87 latidos por minuto. Un estudio
endoscpico practicado 5 horas despus de su
ingreso revela alguna erosin superficial limpia
en el tercio distal del esfago y varias erosiones
superficiales en la zona antral prepilrica, una
de ellas con un punto de hematina en su base.
No quedan restos hemticos en la cavidad gs
trica. El paciente se haba medicado con diclofenaco 150 mg/da los ltimos 8 das por una
ciatalgia. Cul sera la actitud ms recomen
dable?:
1.
2.
3.

4.

5.

6.

Repetir nuevamente la pauta inicial pero mantenida durante 14 das.


Repetir la pauta inicial pero cambiando
Amoxicina por Tetraciclina (500 mg/6h).
Repetir la pauta inicial pero cambiando Claritromicina por Metronidazol (500 mg/8h).
Es necesario practicar cultivo de biopsia gstrica y antibiograma para decidir la combinacin antibitica con mayores garantas de xito.
Administrar Omeprazol (20 mg/12h), Amoxicina (1 g/12h) y Levofloxacino (500 mg/12h)
durante 10 das.

Tratamiento con inhibidores de la bomba de


protones por va oral y alta hospitalaria.
Tratamiento erradicador de Helicobacter pylori de
forma emprica y alta hospitalaria.
Tratamiento hemosttico con sonda de calor de
la erosin con hematina en su base y perfusin
endovenosa de inhibidores de la bomba de
protones durante tres das.
Ayuno total y tratamiento con perfusin endovenosa de inhibidores de la bomba de protones
durante 48 horas.
Colocacin endoscpca de un hemoclip en la
lesin erosiva con signos de hemostasia reciente.

Una mujer de 45 aos, madre de un nio celaco


y sin sntomas ni signos de patologa digestiva,
es evaluada por anemia ferropnica detectada
en analtica rutinaria. Cul de los siguientes
enunciados es cierto respecto a este caso?:
1.
2.

3.

4.

l riesgo de esta mujer de padecer enfermedad


celaca es similar al de la poblacin general.
La ausencia de diarrea hace que se pueda
excluir la enfermedad celaca como causa de la
anemia de esta paciente.
La determinacin del alelo HLA-DQ2, de
resultar positiva, establecera el diagnstico de
enfermedad celaca.
La evaluacin inicial de esta paciente debera
incluir la determinacin de anticuerpos antiendomisio y/o anticuerpos antitransglutaminasa
tisular.
Examen MIR. Versin 0. Enero 2008- PreguntasMIR

5. La edad de la paciente permite excluir la enfermedad celaca del diagnstico diferencial.


7.

Respecto a la enfermedad de Crohn. Cul de


los siguientes enunciados es cierto?:
1.

2.

3.

4.
5.

4,
5.
10.

El proceso inflamatorio est limitado a la


mucosa y a la submucosa superficial, y las capas
ms profundas permanecen respetadas.
Para establecer el diagnstico de enfermedad
de Crohn se precisa la presencia de granulomas no caseificantes.
El diagnstico de enfermedad de Crohn no
debe considerarse en individuos mayores de
50 aos.
El tabaco es un factor protector para la enfermedad de Crohn.
No todos los pacientes con fstulas perianales
presentan signos endoscpicos de inflamacin
en el coln.

Cul de las siguientes afirmaciones relativas al


tratamiento de la hepatitis crnica viral es
FALSA?:
1.

2.

3.

4.
5.

8.

A un paciente de 60 aos, sin antecedentes fami


liares de cncer colorrectal o de poliposis, se le
halla en una colonoscopia, un nico plipo ssil
de 0,5 cm en el sigma que se reseca por comple
to en un solo fragmento en el mismo acto endoscpico. La colonoscopia ha sido completa hasta
el ciego y la preparacin del coln era excelente.
La anatoma patolgica del plipo revela un
adenoma tubular con displasia de bajo grado.
Cul de las siguientes recomendaciones de
seguimiento sera la ms adecuada?:
1.
2.
3.

4.

5.

9.

Programar un nuevo control colonoscpico


dentro de 1 ao.
Programar un nuevo control colonoscpico
dentro de 5 aos.
No hace falta programar una nueva colonoscopia porque el plipo ya ha sido resecado y no
tena displasia de alto grado.
Para determinar con mayor precisin el intervalo de tiempo en el cual hay qe repetir una
nueva colonoscopia es preciso realizar un anlisis de inestabilidad de microsatlites en el
plipo resecado y eventuahnente secuencia-cin
de los genes reparadores del ADN.
Repetir un nuevo control colonoscpico a los
tres meses con biopsias de la cicatriz de la base
de reseccin del plipo.

Un hombre de 28 aos asintomtico es remitido


a consultas por detectarse en analtica del exa
men de rutina laboral una cifras de bilirrubina
total en sangre de 3,8 mg/100 mi, con resto del
perfil sanguneo heptico normal. Refiere cifras
ocasionalmente similares de bilirrubina en ex
menes previos. Qu prueba cree importante
para orientar el diagnstico?:
1.
2.
3.

Colangiopancreatografa endoscpica retrgrada (CPRE) para estudiar la va biliar.


No es preciso hacer ms exploraciones.
Determinacin de bilirrubina total y fraccio-

11.

Una respuesta viral mantenida (negativizacin


del ARN) en la hepatitis crnica C se considera
curacin de la infeccin viral.
Una respuesta viral mantenida (negativizacin
del ADN) en la hepatitis crnica B se considera
curacin de la infeccin viral.
La ausencia de respuesta viral temprana permite interrumpir el tratamiento en pacientes
con hepatitis crnica C, evitando efectos secundarios y costes econmicos innecesarios.
La duracin del tratamiento antivira en la
hepatitis crnica C depende del genotipo viral.
El tratamiento antiviral indicado en la hepatitis
crnica por virus C es la asociacin de interfern alfa pegiado y ribavirina.

Hombre de 60 aos, diagnosticado de cirrosis


heptica virus C, en situacin clnica Child B,
que presenta hemorragia por varices gstricas.
El tratamiento mdico, la terapia endoscpica y
el taponamiento han fracasado, continuando
con hemorragia digestiva por varices. El trata
miento ms adecuado ser:
1.
2.
3.
4.
5.

12.

nada y frotis sanguneo.


Colecistografa oral,
Determinacin de virus hepatotropos.

Trasplante heptico urgente.


Realizacin de una ciruga resectiva: desconexin cigo portal {tcnica de Sugiura).
Realizacin de una portocava calibrada (shunt
portosistmico quirrgico).
Shunt transyugular porto sistmico ntraheptico.
La mejor ciruga derivativa de urgencia es la
derivacin selectiva espleno-renal distal de
Warren.

Un hombre de 52 aos acude a consulta refi


riendo un cuadro de cansancio generalizado,
leve prdida de peso, dolores articulares en
muecas y rodillas y disminucin de la libido.
No refiere hbitos txicos ni ingesta de medica
mentos. A la exploracin llama la atencin un
bronceado de la piel (no relacionado con toma
de sol) y discreta hepatomegaia. En la analtica
realizada destaca: glucosa basal 180 mg/dl;
GOT: 78 U/1; GPT: 89 U/1; ferritina 650 ng/ml;
transferrina: 260 mg/dl; saturacin de transferrina: 66^6%; hierro srico: 220 ug/dl. Qu
otra prueba complementaria entre las siguientes
solicitara inicalmente para llegar al diagnsti
co?:
1.
2.
3.
4.

Prueba de sobrecarga oral a la glucosa.


Estudio gentico de la mutacin C282Y,
Carga viral del virus de la hepatitis C.
Niveles de testosterona en sangre.
Examen MIR. Versin 0. Enero 2008- PreguntasMIR

13.

5. Resonancia magntica de las articulaciones de


la mueca,

colon transverso. Cul es el diagnstico de


sospecha ms probable?:

Ante la presencia de una masa heptica asintomtca descubierta de forma casual, es FALSO:

1.
2.
3.
4.
5.

1.
2.

3.

4.

5.

14.

1.
2.
3.
4.
5.
19.

1.
2.

4.

5.

El paciente explica prdida de apetito.


Se le detecta Mperglicemia en varias analticas
realizadas.
El paciente refiere deposiciones amarillentas y
brillantes con prdida de peso.
Su enfermedad ya lleva muchos aos de evolucin.
El paciente se queja de dolor incapacitante.

2.
3.

5.
20.

En un paciente con pancreatitis aguda que pre


senta mal estado general, leucocitosis de
20.000/uX, fiebre de 39C, insuficiencia renaL,
disnea y shock, la actuacin recomendada es:
1.
2.
3.
4.
5.

Laparotoma de urgencia.
Nutricin parentera total.
Albmina i.v. hasta que la albmina srica
supere los 3 g/L.
Dextrano 60.
Puncin del pncreas con aguja fina y cultivo.

Un paciente de 25 aos inici hace 6 semanas un


cuadro de diarrea con sangre y fiebre por lo
que fue tratado con amoxicilina/clavulnico
durante 7 das sin mejora. Ante la persistencia
de los sntomas se practica una colonoscopia
que demuestra un recto normal y la presencia
de aftas en sigma y lceras longitudinales en

Iniciar tratamiento antibitico y mantener


actitud conservadora.
Cambiar la nutricin parentera por enteral.
Tratamiento quirrgico: desbridamiento o
necrosectoma.
Iniciar tratamiento con un inhibidor de las
proteasas.
Aumentar el tratamiento analgsico.

Un paciente acude a su consulta y usted hace el


diagnstico de presuncin de diverticulitis agu
da, porque relata cuatro signos frecuentes de
esta enfermedad. Uno de los siguientes NO suele
acompaar a la diverticulitis aguda de sigma:
1.
2.
3.
4.
5.

21.

Bacteriemias.
Ulcus duodenal.
Trombosis profunda.
Neumonas.
Meningitis graves.

Qu tenemos que hacer cuando en una pan


creatitis aguda grave se demuestra por el culti
vo, realizado en la puncin-aspiracin con agu
ja fina guiada por TAC abdominal, infeccin de
la necrosis pancretica?:
1.

4.

17.

Litiasis intraheptica.
Parasitosis por Clonorchis Sinensis.
Colangitis esclerosante primaria.
Hemocromatosis.
Enfermedad de Caroli.

Un nio de 10 aos sufri un accidente de bici


cleta y hubo que intervenirlo quirrgicamente
por un hemoperitoneo. Exista una ruptura del
bazo y hubo que practicar una esplenectoma
urgente. Todas las entidades que se refieren a
continuacin son ms frecuentes en este nio
que en la poblacin normal, SALVO una. Sea
le sta:

Cundo sospecharemos que un paciente diag


nosticado previamente de pancreatitis crnica
est desarrollando una insuficiencia pancretica
exocrina grave?:

3.

16.

18.

Una de las siguientes patologas NO es un factor


predisponerte para el desarrollo de un colangiocarcinoma:
1.
2.
3.
4.
5.

15.

El diagnstico diferencial entre masa slida y


qustica se realizar medante ecografa.
Se debe establecer el diagnstico diferencial
entre hiperplasia nodular focal y el adenoma
heptico.
El hemangioma cavernoso se diagnostica por
resonancia con contraste de gadolinio al ser
muy intenso en T2.
El tratamiento de eleccin de la hiperplasia
nodular focal es quirrgico por la posibilidad
de malignidad.
La biopsia heptica de la lesin ser de utilidad para instaurar una terapia no quirrgica.

Colitis por Clostridium Difficih.


Colitis ulcerosa,
Colitis amebiana.
Enfermedad de Crohn.
Colitis isqumica.

Dolor en fosa ilaca izquierda.


Escalofros.
Fiebre.
Cambio en el ritmo intestinal,
Rectorragas.

Gimbernat dijo en el siglo XIX que las enfer


medades que ms comnmente afligen a la
humanidad son sin duda las hernias. Hasta
finales del siglo XX stas eran la causa ms
frecuente de oclusin intestinal. En las ltimas
dcadas la causa ms frecuente de la obstruc
cin intestinal mecnica es:
1.
2.

Siguen siendo las hernias, suponiendo un 60%


de las causas de leo.
Las tumoraciones malignas, que suponen un
60% de las causas de leo.
Examen MIR. Versin 0. Enero 2008- PreguntasMIR

3.
4.

5.

22.

Un hombre de 72 aos de edad con slo antece


dentes de hipercolesterolemia bien controlada,
leve depresin y enfermedad por reflujo gastroesofgico controlada con omeprazol, acude al
Servicio de Urgencias por presentar desde hace
12 horas dolor intenso en fosa iliaca izquierda y
aumento del nmero de deposiciones (hasta
4/da), lquidas pero sin productos patolgicos.
A la exploracin se observa dolor a la palpacin
superficial y profunda en fosa iliaca izquierda,
ruidos presentes y no signos de irritacin peritoneal. La analtica urgente es normal salvo
leucocitosis (16,3 103/uX) con desviacin iz
quierda. Se le realiza un TAC abdominal urgen
te eu el que se describe imgenes sugerentes de
diverticulosis con signos de divertculitis aguda
sin datos de perforacin. Qu actitud terapu
tica considera ms correcta?:
1.
2.
3.
4.

5.

23.

El sndrome adherencia! postoperatorio, que


supone el 60% de las causas de leo.
Los vlvulos e invaginaciones, que sumados
en nios y adultos suponen un 60% de las causas de leo.
Los sndromes metablicos tipo diabetes y
otros, lo que sumado a la pluripatologa y ancianidad de los pacientes quirrgicos actuales
suponen un 60% de las causas de leo.

Realizacin de colonoscopia urgente.


Colocacin urgente de un drenaje percutneo,
dirigido por TAC, en rea inflamada.
Intervencin quirrgica urgente para extirpar
zona del colon inflamada.
Alta domiciliaria, recomendando dieta rica en
fibra y que acuda nuevamente a urgencias si
empeora la situacin del paciente.
Observacin hospitalaria con dieta absoluta y
administracin de ciprofloxacino y metronidazol por va i.v.

25.

1.
2.
3.
4.
5.
26.

27.

Estenosis artica.
Taponamiento cardiaco.
Pericarditis constrictiva.
Embolia pulmonar.
Enfisema pulmonar (cor pulmonale).

Est contraindicado usar betabloqueantes como


tratamiento en sujetos hiperensos que tengan
adems una de las siguientes circunstancias
acompaantes, SALVO:
1.
2.
3.
4.
5.

Bloqueo aurculo-ventricular de 2o grado.


Asma bronquial,
Enfermedad del nodulo snusal.
Diabetes mellitus en tratamiento con hipoglucemantes.
Insuficiencia cardiaca.

2.
3.
4.
5.
29.

Digitalicos.
Inhibidores de la ECA.
Antagonistas de Calcio.
Betabloqueantes.
Diurticos.

En relacin con el empleo de la trombolisis en el


infarto agudo de miocardio, seale la afirma
cin FALSA:
1.

24.

Derrame pericrdico.
Prolapso de un festn de la vlvula mitra!.
Insuficiencia mitral moderada.
Absceso periartico.
Insuficiencia artica severa.

Un paciente de 63 aos, con Insuficiencia Car


diaca por cardiopata hipertensiva, en situacin
estable (en clase funcional I segn grado de
disnea), presenta en el Ecocardiograma, Disfun
cin Sistlica (Fraccin de Eyeccin < 35%).
Qu grupo de frmacos estara ms indicado
como tratamiento inicial?:
1.
2.
3.
4.
5.

28.

Miocardiopata restrictiva.
Pericarditis crnica constrictiva.
Miocardiopata dilatada.
Miocardiopata hipertrfica obstructiva.
Cor pulmonale crnico.

Cnl de los siguientes hallazgos ecocardiogrfeos es un criterio mayor en el diagnstico de la


Endocarditis Infecciosa?:
1.
2.
3.
4.
5.

En cul de las siguientes situaciones clnicas, el


pulso paradjico (disminucin de la presin
arterial de ms de 10 mraHg durante la inspira
cin), no est presente en la exploracin fsica?:
1.
2.
3.
4.
5.

Hombre de 55 aos, fumador de 20 cigariUos/da y bebedor de ms de 90 gramos de alco


hol/da. Ingresa por presentar en los ltimos 2
meses disnea progresiva hasta hacerse de repo
so, ortopnea y crisis de disnea paroxstica noc
turna. En la exploracin destaca soplo pansistlico, crepitantes de gruesa burbuja diseminados
y edemas en extremidades inferiores. Electro
cardiograma: fibrilacin auricular con respues
ta ventricular a 130 Ipm y bloqueo completo de
rama izquierda. En la radiografa de trax pre
sentaba cardiomegalia global, derrame pleural
bilateral, edema intersticial en bases y lneas B
de Kerley. Cul sera su primer diagnstico de
sospecha?:

Es til para disminuir el tamao de la zona


infartada.
Es til para disminuir las arritmias.
Es til para disminuir la mortalidad.
Es til para limitar la disfuncin ventricular
izquierda.
Slo es til en las primeras horas post-nfarto.

El mecanismo principal de accin de la tenecteplasa (TNK) en los sndromes coronarios agu


dos con elevacin del segmento ST es:

Examen MIR. Versin 0. Enero 2008- PreguntasMIR

1.
2.
3.
4.
5.

Estimular el complejo protrombinasa y la


formacin de trombina.
Estimular la conversin de plasmingeno en
plasmina.
Bloquear el receptor GP Hb/EIa de las plaquetas.
Inhibir la formacin de trombina y plasmingeno.
Estimular la produccin de heparinodes.

3.
4.
5.
34.

Seale la respuesta INCORRECTA respecto a


la estenosis artica:
1.
2.

30.

Paciente de 65 aos de edad, con antecedentes


de haber sufrido un infarto de miocardio hace
dos aos. Acude aj Hospital por un cuadro de
malestar general, sudoracin y palpitaciones.
En el electrocardiograma se observa un ritmo
regular a 170 latidos por minuto con complejos
QRS de 0.14 segundos. Cul sera su primer
diagnstico?:
1.
2.
3.
4.
5.

31.

Fibrilacin auricular paroxstica.


Taquicardia nodal.
Taquicardia ventricular.
Taquicardia paroxstica supraventricular.
Flutter auricular.

3.

4.
5.

35.

Una de las siguientes respuestas, referidas a la


arritmia cardiaca, fibrilacin auricular, es
FALSA:
1.
2.
3.

4.

5.

Deben descartarse factores precipitantes como


hipertiroidismo o embolismo pulmonar.
La anticoagulacin est indicada en todos los
casos.
En situaciones de inestabilidad hemodinmica
es adecuado el tratamiento de cardioversin
elctrica.
Cuando el dimetro de la aurcula izquierda es
mayor de 45 mm, puede ser difcil revertir a
ritmo sinusal.
Cuando el dimetro de la aurcula izquierda es

5.

ritmo sinusal, si se consigue la reversin.


32.

Se considera indicacin de desfibrilador im


pla ntable:
1.
2.
3.
4.

5.

33.

37.

Extrasstoles ventriculares complejos sintomticos en paciente sin cardiopata.


Taquicardia ventricular incesante.
Fibrilacin ventricular que ocurre en la primera
hora tras un infarto agudo de miocardio.
Parada cardiaca por fibrilacin ventricular en
paciente con infarto de miocardio crnico anterior extenso.
Sncopes de repeticin en paciente con estudio
cardiolgico normal.

2.
3.

4.
5.

38.
1.
2.

Tirotoxicosis.
Embarazo.

Rabdomioma.
Lipoma.
Fibroma.
Mixoma.
Hemangioma.

Seale la respuesta correcta sobre la ciruga de


la trombosis venosa:
1.

En cul de estas posibles causas desencadenan


tes de Insuciencia Cardiaca NO existe incre
mento del gasto cardiaco?:

Embolia de pulmn.
Derrame pericrdico severo.
Neumotorax espontneo.
Infarto de miocardio con embolia a la arteria
radial izquierda.
Diseccin artica.

Cul es el tumor benigno cardiaco ms fre


cuente en adultos?:
1.
2.
3.
4.
5.

mayor de 45 mm. puede ser difcil mantener el

Suele requerir tratamiento quirrgico en personas mayores.


La edad no suele ser una contraindicacin para
el recambio valvular.
La muerte sbita es una complicacin poco
comn de los pacientes con estenosis artica
sintomtica.
Cuando la estenosis artica se hace sintomtica est indicada la sustitucin valvular.
Los pacientes con estenosis artica que desarrollan angina tienen un elevado riesgo de
mortalidad.

Cul de los siguientes diagnsticos es ms pro


bable en un paciente de 33 aos con anteceden
tes familiares de muerte sbita que acude por
dolor torcico intenso, de inicio brusco y en
reposo y que en la exploracin presenta un so
plo diastlico precoz y ausencia del pulso radial
izquierdo?:
1.
2.
3.
4.

36.

Anemia.
Fiebre.
Taquiarritmia.

Cuando afecta al cayado de la safena interna


puede ligarse ste por debajo del trombo y evitarse as la anticoagulacin.
La anticoagulacin es obligatoria cuando se
practica la extirpacin del sistema superficial.
El tratamiento quirrgico de las varices de
miembros inferiores requiere el ingreso hospitalario de los pacientes.
El edema y las lceras son complicaciones
poco frecuentes de las varices.
No es bueno administrar antiinflamatoros
tpicos en las varices superficiales.

La alteracin ms frecuente de las pruebas de


funcin pulmonar en los supervivientes del Sn
drome de Distrs Respiratorio del Adulto es:
Examen MIR. Versin 0. Enero 2008- PreguntasMIR

1.
2.
3.
4.
5.

pausas respiratorias durante el sueo. El registro


muestra un ndice de apnea-hipopnea de 4/hora a
expensas de apneas obstructivas, una Sa02 media
durante el sueo de 94% y una Sa02 mnima de
$6%. El diagnstico es:

Disminucin de la complianza pulmonar.


Disminucin de la capacidad de difusin.
Aumento de resistencia de la va area.
Hipertensin pulmonar.
Disminucin de la capacidad pulmonar total.

1.
39.

La siguiente gasometra arterial: PH 7.40, Pa02


98, PaC02 38, HC03 25, refleja:
1.
2.
3.
4.
5.

2.

Acidosis metablica.
Alcalosis metablica.
Valores normales.
Alcalosis respiratoria.
Acidosis respiratoria.

3.

4.
5.

40.

En el asma persistente leve. Cul de los si


guientes esquemas teraputicos puede conside
rarse de eleccin?:
1.
2.
3.
4.
5.

41.

2.

3.

4.
5.

42.

Neumonitis por hipersensibilidad.


Neumona organizada criptognica.
Fibross pulmonar dioptica.
Neumona intersticial no especfica.
Neumona eosinfla crnica.

A un paciente obeso y roncador intenso, sin


somnolencia diurna, con antecedentes de insufi
ciencia cardiaca, se le practica una polisomnografa despus de haber detectado su pareja

El hallazgo ms frecuente en la radiografa de


trax en un paciente con asma es:
1.
2.
3.
4.
5.

45.

Por alteraciones en la relacin "ventilacin/perfusin".


Por vaciado incompleto de unidades alveolares
a causa del aumento de la frecuencia respiratoria.
Por vaciado incompleto de unidades alveolares
a causa de una disminucin en la "orden respiratoria,\
Por los problemas mecnicos asociados a una
alcalosis metablica.
Por los problemas mecnicos asociados a una
alcalosis respiratoria.

Un paciente de 60 aos, fumador, con historia


de disnea de un ao de evolucin presenta una
auscultacin de crepitantes y se comprueban
acropaquias. La TC torcica demuestra una
afectacin reticular en la periferia del pulmn.
El diagnstico ms probable es:
1.
2.
3.
4.
5.

43.

Salmeterol a demanda.
Salbuamol pautado ms salbutamol a demanda.
Budesonida a demanda ms salbutamol a demanda.
Budesonida pautada ms salbutamol a demanda.
Budesonida pautada ms salbutamol pautado.

Por qu mecanismo aparece la "hiperinsuflacin dinmica" durante el ejercicio en los en


fermos con EPOC?:
1.

44.

2.

3.
4.
5.

El paciente puede ser dado de alta con tratamiento antibitico y realizar una radiografa de
control en un mes.
Se debe realizar un estudio de tomografa
computarzada (TC) torcica ante la sospecha
de neoplasia pulmonar.
El paciente puede ser bacilfero y se debe
realizar un anlisis del esputo.
Se debe obtener un diagnstico citolgco
mediante broncoscopia o puncin percutnea.
Se debe completar el estudio con una Resonancia Magntica con gadolinio.

Cul sera la primera exploracin radiolgica


que se hara en un paciente con dolor torcico?:
1.
2.
3.
4.
5.

47.

Hiperinsuflacin pulmonar.
Condensaciones alveolares bilaterales y difusas.
Radiografa de trax normal.
Engrasamiento de paredes bronquiales.
Neumomediastino.

Un paciente de 43 aos acude a urgencias por


disnea, tos y fiebre de una semana de evolucin y
en la radiografa de trax presenta una cavidad de
paredes lisas en lbulo superior derecho rodeada
por reas de consolidacin alveolar. Cul es la
conducta inicial ms correcta?:
1.

46.

Se trata de un roncador simple que no precisa


tratamiento.
Se trata de un sndrome de apnea del sueo
subsidiario de tratamiento con CPAP nasal.
Se trata de un sndrome del sueo subsidiario
de dieta hipocalrica y dormir en decbito lateral.
Se trata de un sndrome de apneas centrales
secundarias a la insuficiencia cardiaca.
Los resultados de polisomnografa no son
compatibles con la historia clnica.

Rx PA y Lat del trax en inspiracin.


Rx PA y Lat del trax en espiracin.
Rx en decbito supino.
Rx oblicuas de ambos pulmones.
Rx en proyeccin ordtica.

Mujer de 55 aos, postmenopasica y fumadora


activa, que consulta por dolor de hombro iz
quierdo irradiado a brazo de varias semanas de
duracin. En la exploracin fsica destaca ptosis
y miosis del ojo izquierdo. Cul sera su acti
tud?:
Examen MIR. Versin 0. Enero 2008- PreguntasMIR

1.
2.
3.
4.
5.
48.

cia de adenopatias mediastnicas afectadas por


tumor. El paciente es operable, Cul de estas
actitudes le parece ms adecuada llegado este
momento?:

Solicitar estudio de osteoporosis.


Solicitar estudio oftalmolgico.
Realizar anticuerpos antireceptor de acetilcolina.
Solicitar TAC cerebral.
Solicitar radiografa simple de trax.

1.

Mujer de 70 aos que acude a urgencias por


cuadro sincopal. Intervenida de neoplasia de
coln hace 2 semanas. En la exploracin: pre
sin venosa a 4 cm sobre horizontal, 102 latidos
por minuto, 35 respiraciones por minuto. T:
37,8 C, dolorimiento abdominal difuso, resto de
exploracin sin datos patolgicos; pulsioximetria: saturacin de oxgeno basal: 86%. ECG:
ritmo sinusal a 100 Ipm, inversin de ondas T de
VI a V4. Radiografa de trax: elevacin de
hemidiafragma izquierdo con pinzamiento seno
costofrnico. Cul es el diagnstico ms pro
bable?:

2.

3.

4.
1.
2.
3.
4.
5.

Infeccin respiratoria.
Infarto de miocardio con Sndrome de Dressler.
Taponamiento cardaco.
Tromboembolismo pulmonar masivo.
Dehiscencia de sutura con distress respiratorio.
5.

49.

Paciente de 28 aos con cuadro de dos das de


evolucin de tos y fiebre de 39. En la radiogra
fa de trax se observa la existencia de un de
rrame pleural que ocupa la mitad del hemitrax
izquierdo. Los hallazgos bioqumicos de la toracocentesis son los siguientes: pH 7,10, LDH
1200 u/L, protenas 4,2, ADA 70. Cul ser la
conducta ms apropiada a seguir?:
1.

2.

3.

4.
5.

Iniciar tratamiento antibitico emprico y


esperar los resultados del cultivo del lquido
pleural para colocar un tubo de drenaje torcico.
Colocar un drenaje pleural, instilar fbrinolticos intrapeurales e iniciar tratamiento
antibitico emprico.
Iniciar tratamiento antibitico emprico y
repetir la toracentesis al cabo de 24 horas para
valorar la actitud a seguir en funcin de la
evolucin de los parmetros bioqumicos del
lquido pleural.
Colocar un tubo de drenaje torcico e iniciar
tratamiento antibitico emprico.
Efectuar una broncoscopia para toma de muestras microbiolgicas, colocar un tubo de drenaje torcico e iniciar tratamiento antibitico
emprico.

51.

Un paciente ingresado despus de un politraumatismo grave indica a las 24 horas disnea pro
gresiva. Una primera radiografa de trax
muestra infiltrados alveolares bilaterales que,
pasadas unas horas, evolucionan a una imagen
de "pulmn blanco". Cul es su diagnstico?:
1.
2.
3.
4.
5.

52.

Paciente de 62 aos, fumador importante, que


presenta masa en lbulo inferior derecho con
diagnstico de carcinoma no microctico de
pulmn que infiltra la grasa mediastnica y
rodea al esfago, desplazndolo. No hay eviden-

Tromboembolismo pulmonar secundario a la


inmovilizacin.
Contusin pulmonar.
Sndrome de distres respiratorio del adulto.
Hemorragia alveolar.
Derrame pleural bilateral masivo por hemotrax.

En relacin con la patologa del nervio ptico y


los nervios oculomotores, es cierto:
1.
2.
3.

50.

El paciente presenta un carcinoma broncognico no microctico estadio IV (T4N0M0), por


lo que no es resecable. Se debe tratar con quimioterapia y radioterapia.
El paciente presenta un carcinoma broncognico no microctico estadio clnico IECB
(T4N0M0). La realizacin de una ecoendoscopia esofgica que determine el grado de
afectacin de la pared del esfago sera til
para plantear una posible extirpacin, aunque
se debe empezar con quimioterapia y radioterapia.
El paciente presenta un carcinoma broncognico no microctico estadio clnico IHB
(T4N0M0). Se debe operar, extirpando todo el
pulmn y la totalidad del esfago.
El paciente presenta un carcinoma broncognico no microctico estadio clnico fV
(T4N0M0). La realizacin de una ecoendoscopia esofgica que determine el grado de
afectacin de la pared del esfago sera til
para plantear una posible extirpacin, aunque
se debe empezar con quimioterapia y radioterapia.
El paciente presenta un carcinoma broncognico no microctico estadio clnico IHB
(T4N0M0), por lo que en ningn caso es resecable. Se debe administrar tratamiento paliativo con quimioterapia.

4.
5.

Un infarto de la corteza occipital produce una


hemianopsia homnima ipsilateral.
La duracin ms habitual de la amaurosis
fugaz es de entre 45 y 60 minutos.
En la neuritis ptica las pupilas son isocrcas
y puede encontrarse un defecto pupar aferente.
La prdida de agudeza visual en el edema de
papila es muy grave.
En las lesiones compresivas del III par (motor
Examen MIR. Versin 0. Enero 2008- PreguntasMIR

ocular comn) hay caractersticamente un respeto de la funcin pupilar.

4.
5.

53.

En qu situacin estara contraindicada la


fibrinolisis endovenosa para tratar un infarto
cerebral?:
1.
2.
3.
4.
5.

54.

57.

Edad superior a setenta aos.


Evolucin de la clnica de ms de dos horas.
Historia de tratamiento hipotensor.
Mejora espontnea del dficit neurolgico.
TC cerebral normal.

En un enfermo con una clnica de prdida de


fuerza progresiva de 48 horas de evolucin que
se inici a nivel distal, y ha ido ascendiendo
progresivamente, una de las siguientes afirma
ciones es FALSA:
1.
2.

En un paciente que presenta un cuadro agudo


de cuadro confusiona], oftaimoparesia por afec
tacin del sexto par bilateral y ataxia de la mar
cha pensara en:

3.

4.
1.
2.
3.
4.
555.

5.

58.

En un periodo de 3 meses, un hombre de 42


aos ha presentado cuatro episodios de desco
nexin de su entorno, asociados con postura
distnica de la mano derecha y movimientos de
masticacin, de un minuto de duracin, que
dando a continuacin confuso y con dificultad
para la expresin verbal durante 10 minutos.
Despus se recupera con normalidad pero no
recuerda lo que le ha ocurrido. Cul es la acti
tud ms correcta?:
1.
2.
3.

4.
5.

56.

Encefalopata hperglucmca.
Encefalopata de Korsakoff.
Infarto cerebeloso.
Administrarle inmediatamente tiamina.
Intoxicacin por plomo.

Iniciar tratamiento con clobazam oral y realizar


un electroencefalograma (EEG).
Iniciar tratamiento con carbamacepina y realizar
una resonancia magntica (RM) y un EEG.
Realizar un EEG y una RM de forma ambulatoria y esperar los resultados antes de iniciar el
tratamiento.
Ingresar en la Unidad de Cuidados Intensivos e
iniciar tratamiento con fenitoina intravenosa.
Recomendar observacin por su familia y
volver a revisin en tres meses.

Una joven de 13 aos sin antecedentes persona


les de inters, presenta bruscamente mientras
jpasea con sus padres por un centro comercial,
cuadro vertiginoso asociado a vmitos. Inme
diatamente despus, refiere parestesias en
hemicuerpo derecho y disartria, que remiten en
pocos minutos, y a continuacin cefalea, que
mantiene a su llegada al centro de salud. Seale
la respuesta correcta:
1.
2.
3.

El diagnstico ms probable es el de sncope


vasovagal.
Si la exploracin fsica es normal no se requieren ms estudios.
La ausencia de antecedentes personales excluye el diagnstico de migraa.

2.

3.

4.
5.

El sexo de la paciente hace improbable un


diagnstico de Miastenia Gravis.
La presencia de lesiones desmieliizantes en la
RMN cerebral confirmara que se trata de
Miastenia Gravis.
En pacientes de Miastenia Gravis con afectacin
ocular limitada, se observa presencia de Ac
antireceptores de acetilcolina en slo el 50%.
Este dato positivo confirmara el diagnstico.
Se trata de un sndrome de Horner y la Rx de
trax mostrara un tumor de Pancoast.
Los inmunosupresores no han demostrado
eficacia en la Miastenia Gravis.

En una paciente de 68 aos con dolor facial en


el que se sospecha una neuralgia idioptica de la
segunda rama del nervio trigmino derecho,
una de las siguientes respuestas NO sera
correcta:
1.
2.
3.
4.
5.

60.

Sospechara un sndrome de Guillain-Barr,


Vigilara la aparicin de complicaciones por
neuropata autnoma.
Esperara encontrar disociacin abminacitolgica a partir de los 10 das desde el inicio
del cuadro.
Indicara tratamiento con Inmunoglobulinas
intravenosas.
Los hallazgos neurofsolgicos aparecen antes
de que sea evidente la clnica.

Una mujer de 34 aos refiere diplopia de varias


semanas. Unos meses antes haba sufrido diplo
pia, que remiti espontneamente. La explora
cin muestra leve ptosis palpebral:
1.

59.

El empleo de analgsicos y antiemticos no


son aconsejables en este supuesto.
Se debe evaluar la existencia de estmulos
desencadenantes.

Hpoestesia en la zona malar derecha.


La duracin del dolor es de unos pocos segundos.
Los paroxismos dolorosos pueden desencadenarse al tocar suavemente una zona facial.
El reflejo corneal es normal.
Indicara tratamiento con carbamacepina.

Acude a urgencias un paciente de 80 aos que


presenta desde hace 2 das alteracin del nivel
de conscienca de forma fuctuante con lenguaje
incoherente, agitacin y desorientacin. La
familia refiere que durante la ltima semana al
paciente se le ha prescrito lormetazepam por
dificultad para conciliar el sueo. En la expo-

Examen MIR. Versin 0. Enero 2008- PreguntasMIR

racin fsica el paciente se encuentra afebril con


TA 140/70 y sin focalidad neurolgica motora ni
sensitiva, con lenguaje incoherente y agitado. La
frecuencia cardiaca era rtmica a 65 Ipm. Cul de
las afirmaciones le parece correcta?:
1.

2.

3.

4.
5.

61.

La esclerosis mltiple es un proceso inflamato


rio y desmielinizante del SNC. En el diagnstico
de esta enfermedad es muy til la presencia de:
1.
2.
3.
4.
5.

62.

65.

66.

67.

Adenoma hipofsario no funcionante.


Prolactinoma.
Meningioma del tubrculo solar.
Pinealoma.
Intoxicacin por benzodiacepinas.

Levotiroxina a dosis susttutivas y reevaluar a

2.

Levotiroxina a dosis supresoras y reevaluar a


los 3 meses.
Tiroidectoma total.
Administrar Yodo131.
Simplemente observar y repetir ecografa
tiroidea a los 6 meses para valorar tamao.

Mujer de 88 aos, natural del Pirineo, donde


siempre ha vivido, que consulta por un gran
bocio multinodular junto con arritmia y hallaz
go de un hipertiroidismo por T3 (T4 libre nor
mal, T3 libre 8 praol/L, normal de 4 a 6.8, y
TSH de 0.15 mUL, normal de 03 a 5). Reconoce
que su bocio existe desde hace ms de 30 aos
sin claro crecimiento reciente. No refiere disfagia ni estridor. Lleva tratamiento para hiper
tensin arterial, controlada con hidroclorotiacida y potasio, as como broncodilatadores y corticoides inhalados por una EPOC. Cul ser,
probablemente, la actitud teraputica ms indi
cada?:
No tratar, ya que la T4 libre es normal.
Hemitiroidectoma ms ismectoma.
Iniciar L-tiroxina sdica.
Iniciar propranolo!.
Tratamiento con yodo radioactivo.

En una paciente de 40 aos se encuentra una


cifra de calcio de 11,8 mg/dl (normal menos de
10,5 mg/dl) en un anlisis realizado por otro
motivo. La determinacin de PTH solicitada en
el estudio de la hipercalcemia arroj una cifra
de 115 pg/ml (lmite superior de la normalidad,
45 pg/ml). La densidad sea, el aclaramiento de
creatinina y la calciuria son normales. Cul
debe ser la conducta a seguir?:
1.
2.
3.
4.
5.

68.
Angoma venoso.
Malformacin arterovenosa.

1.

1.
2.
3.
4.
5.

Cul de las siguientes lesiones vasculares cere


brales conlleva un menor riesgo de sangrado
intracraneal?:
1.
2.

Qu actitud tomara ante un hombre de 25


aos con un nodulo tiroideo indoloro, de 4 cm.
de tamao, de reciente aparicin, fro en la
gammagrafa, con estudio hormonal normal, y
con abundantes clulas foliculares en la puncin
aspiradora con aguja fina?:

3.
4.
5.

Elevaciones del cido rico en plasma.


Bandas oligoclonales en el lquido cefalorraqudeo.
Ms de 100 linfocitos por microlitro en el
lquido cefalorraqudeo.
Cifras elevadas de cidos grasos de cadena
muy larga en plasma.
Ms de 50 polimorfonucleares por microlitro
en el lquido cefalorraqudeo.

Omeprazol.
Cisapride.
Risperidona.
Clozapina.
Quetiapina.

Cavernoma.
Aneurisma mictico de la arteria cerebral
media.
Aneurisma sacular de la arteria comunicante
anterior.

los 6 meses.

Ante un cuadro clnico de amenorreagalactorrea y prdida de campo visual el primer


diagnstico a considerar es:
1.
2.
3.
4.
5.

64.

5.

Cul de los siguientes frmacos cree que tiene


ms posibilidades de inducir un parkinsonismo
yatrgeno?:
1.
2.
3.
4.
5.

63.

Se trata de un paciente que ha sufrido un accidente vascular cerebral y debe realizarse una
TAC cerebral urgente y administrar benzodiacepinas para controlar la agitacin.
Se trata de una clnica sugestiva de crisis comcia y debe realizarse un electroencefalograma urgente y administrar benzodiacepinas
endovenosas.
Se trata de un delirium y debe retirarse el
lormetazepan y administrar un neurolptico
para controlar la agitacin.
Debera realizarse una puncin lumbar para
descartar posible etiologa infecciosa.
Se trata de un efecto indeseable del lormetazepam que desaparecer al cabo de unos das sin
ser necesario retirar medicacin.

3.
4.

Administracin de bisfosfonatos.
Paratiroidectoma.
Administracin de quejantes del calcio.
Observacin.
Administracin de diurticos perdedores de
calcio (frosemida).

Ante un paciente, con antecedentes de haber


recibido hace 5 aos radioterapia sobre hipfi
sis, que llega a Urgencias estuporoso, con preExamen MIR. Versin 0. Enero 2008- PreguntasMIR

sin arterial sistlica de 70 mm Hg, fiebre de 39 C


y con sudoracin fra, lo primero que debe hacerse
es:

1.

1.

2.

2.
3.
4.
5.
69.

70.

3.
4.
5.

2.
3.
4.
5.

La acarbosa aumenta la secrecin de insulina.


Las tiazolidindionas bloquean la a-glucosidasa
intestinal.
La metformina reduce la produccin heptica
de glucosa.
No pueden asocian-e a la administracin de
insulina.
Con glibenclamida el riesgo de hipoglcemias
es mnimo.

5.

74.

75.

2.
3.
4.
5.

Complicaciones neoproliferativas especialmente los osteosarcomas de huesos largos.


Complicaciones cardiovasculares.
Complicaciones neoproliferativas especialmente la neoplasia de pulmn.
Los aneurismas intracraneales se observan
hasta un 70% de los casos.
A pesar de las complicaciones asociadas, la
mortalidad no est aumentada en los casos de
acromegalia no tratada.

Si una mujer de 35 aos consulta por presentar


un microprolactinoma de 9 mm, podremos de-

Aumento de la TSH.
Aumento de la CPK (Creatinfosfoquinasa).
Aumento de las transaminasas.
Mialgias.
Artralgias.

En relacin al aumento de la prevalencia de la


obesidad y la inactividad fsica en la poblacin,
se ha incrementado la frecuencia de un trastor
no denominado sndrome metablico. Seale
cual de las siguientes caractersticas NO forma
parte del sndrome metablico:
1.
2.
3.
4.
5.

77.

Pancreatitis aguda grave.


Fstula yeyunal.
Reseccin subtotal de intestino delgado
Expectativa de ayuno de menos de 5 das.
Postoperatorio de hemicolectomia.

Cul de los siguientes NO es un efecto secun


dario frecuente del tratamiento con esta tinas?:
1.
2.
3.
4.
5.

76.

Si permanece asintomtica y no presenta deseo


gestacional no requiere tratamiento.
S permanece asintomtica no requiere tratamiento ni seguimiento/observacin.
An estando asintomtica debe iniciar tratamiento con agonistas dopaminrgicos.
El tratamiento quirrgico es recomendable de
entrada en lesiones prximas a los 10 mm.
Si presenta sntomas, la radioterapia es una
alternativa de tratamiento dado que se normalizan las cifras de prolactina en un 80% de los
casos.

Cul de las siguientes contraindica la nutricin


parenteral?:
1.
2.
3.
4.
5.

Proximal, bilateral, simtrica, dolorosa y motora


Distal, bilateral, simtrica y motora.
Distal, unilateral y motora.
Distal, bilateral, simtrica y sensitiva.
Proximal, bilateral, simtrica y sensitiva.

En relacin con las complicaciones que pueden


acompaar a la acromegalia. Cul es la verda
dera?:
1.

73.

4.

La afectacin ms frecuente en la polineuritis


diabtica es:
1.

72.

3.

Carcinoma medular de tiroides.


Tumor endocrino del pncreas.
Hiperparatiroidismo Primario.
Feocromocitoma.
Adenoma hipofisario.

En relacin con los hipoglicemiantes orales es


cierto que:
1.
2.

71.

Practicar cultivos (hemocultivos, urocultivos)


para localizar el foco infeccioso.
Realizar un test corto de Nuvacthen (ACTH)
intravenoso.
Colocar una va con cloruro sdico al 0,9% e
hidrocortisona.
Colocar una va con cloruro sdico al 0,9%,
glucosa al 10% e hidrocortisona.
Administrar hidrocortisona y tiroxina.

Cul es la enfermedad endocina ms frecuente


en la Neoplasia Endocina mltiple tipo 1 ?:
1.
2.
3.
4.
5.

cirle que:

Hipertrigliceridemia.
Hipertensin arterial.
Hiperglucemia en ayunas
Aumento del permetro de la cintura abdominal
Aumento de la concentracin plasmtica de
colesterol-LDL.

Cul de las siguientes afirmaciones que refle


jan las caractersticas clnicas de la artritis
reumatoide es correcta?:
1.
2.

3.
4.
5.

La afectacin de la columna lumbar es ms


frecuente que la cervical.
La afectacin de las articulaciones interfalngicas distales es ms frecuente que las proximales.
La artritis reumatoide es ms frecuente en los
hombres en reas rurales.
Es una poliartritis de ms de 6 semanas de
duracin.
La artritis reumatoide del adulto se asocia con
un aumento de la frecuencia de uveitis.
Examen MIR. Versin 0. Enero 2008- PreguntasMIR

3.
78.

Paciente de 40 aos eon febrcula y sndrome


constitucional (astenia, anorexia, prdida de
peso), que presenta epistaxis recurrente con
dolor en tabique nasal. Se realiza una radiogra
fa de trax en la que se aprecian infiltrados
pulmonares nodulares cavitados bilaterales. Se
practica una analtica con estos resultados:
12.000 leucos/mm3 con 68% neutrflos (7.800
por mm3), 23% Hnfocitos (2760 por mm3), 5%
monocitos (600 por mm3) y 1% eosinflos (120
por mm3), creatinina normal, anticuerpos anti
citoplasma de neutrfilo positivos (c-ANCA).
Qu entidad sospechara en este paciente?:
1.
2.
3.
4.
5.

79.

2.
3.
4.
5.

84.

Hemoglobina.
Recuento de leucocitos.
Hierro srico.
Alfa 2 globulinas.
Velocidad de sedimentacin globular.

Cul de estas afirmaciones es FALSA con res


pecto a las artritis inducidas por microcristales?:
1.

2.

Pueden ser producidas por cualquiera de los


siguientes cristales: urato monosdico, pirofosfato calcico, hidroxiapatita calcica y oxalato calcico.
El depsito de cristales puede producir cua
dros clnicos similares a la artritis reumatode
o la espondilitis anquilosante.

2.
3.
4.
5.
85.

Artrocentesis, examen del lquido sinovial con


microscopio de luz polarizada y test de Gram
urgente.
Tratamiento con antibiticos intraarticulares.
Estudio inmunolgico, incluyendo factor reumatode.
Lavado articular y tratamiento con antibiticos
intravenosos.
Artrocentesis e infiltracin con corticosteroides de accin prolongada.

Una mujer de 84 aos acude a nuestra consulta


por dolor crnico, a veces intenso, en la zona
media de la columna vertebral torcica, que
aumenta al sentarse y levantarse. A la explora
cin se detecta una marcada cifosis y contractura de la musculatura paravertebral. Tambin se
palpa una masa pulstil en el epigastrio. Cul
es la causa ms probable del dolor?:
1.

Ante un paciente con sospecha de Poliarteritis


Nodosa (PAN). Cul de las siguientes pruebas
complementarias diagnsticas tiene mayor ren
dimiento?:
Complemento srico.
Arteriografa abdominal
Determinacin de anticuerpos anticitoplasma
del neutrfilo (ANCA).
4. Endoscopia digestiva alta.
X Medida del flujo lagrimal.

Una mujer de 58 aos con antecedentes de


diabetes mellitus tipo 1 y artrosis de ambas
rodillas acude al servicio de urgencia por fiebre
de hasta 39C junto con dolor y tumefaccin de
24 horas de evolucin en la rodilla derecha.
Cul de las siguientes actitudes o exploraciones
complementarias est ms indicada?:
1.

La inflamacin ocular (uveitis, conjuntivitis).


La leucopenia.
La hipertensin arterial,
La presencia de afectacin gstrica.
La afectacin renal.

1.
2.
3.

82.

83.

Poliarteritis Nodosa clsica.


Granulomatosis alrgica de Churg-Strauss.
Granulomatosis de Wegerter.
Arteritis de Takayasu.
Artritis Reumatode.

En una paciente de 68 aos con arteritis de


clulas gigantes. Cul de ios siguientes datos de
laboratorio suele estar menos alterado?:
1.
2.
3.
4.
5.

81.

5.

Cul de las siguientes manifestaciones es com


partida por las espondiloartropatias?:
1.
2.
3.
4.
5.

80.

4.

Los cuadros clnicos producidos son especficos para cada uno de los tipos de cristales depositados.
Para hacer el diagnstico es imprescindible el
estudio del lquido sinovial con microscopio
de luz polarizada para identificar el tipo de
cristales.
El lquido sinovial suele ser de tipo inflamatorio, aunque en ocasiones existen cristales en
ausencia de inflamacin,

Aneurisma abdominal con erosin vertebral


acompaante.
Estenosis de canal lumbar.
Hernia de disco aguda.
Osteoporosis con aplastamiento vertebral.
Enfermedad de Paget

Paciente de 40 aos que acude al Servicio de


Urgencias tras sufrir un traumatismo directo
sobre el brazo izquierdo al caerse de la motoci
cleta. La exploracin clnica es la siguiente:
dolor, movilidad anormal e impotencia funcio
nal en la zona media del brazo, pulsos distales
humeral y radial presentes, imposibilidad para
realizar la extensin activa de la mueca y de
dos, e hipoestesia en la zona dorsal del primer
dedo y del primer espacio interdigital. Las ra
diografas muestran una fractura conminuta del
tercio medio del hmero. Cul es la lesin
asociada ms probable que presenta?:
1.
2.
3.

Lesin arteria humeral y nervio mediano.


Lesin nervio cubital.
Lesin nervio radial.
Examen MIR. Versin 0. Enero 2008- PreguntasMIR

4.
5.
86.

Un chico de 20 aos, jugando al ftbol hace una


semana, sufri una torsin en su rodilla al cla
var los tacos de la bota en el csped y girar
bruscamente el tronco. A la exploracin no se
aprecia derrame articular, la palpacin del
tercio posterior de la interlnea articular medial
es dolorosa, duele al forzar la flexin, la manio
bra de Lachman es negativa y los bostezos a
varo y valgo son indoloros. Cul es el diagns
tico ms probable?:
1.
2.
3.
4.
5.

87.

3.

4.
5.

89.

Lesin meniscal del menisco medial.


Lesin capsular pstero-medial.
Lesin del ligamento cruzado anterior.
Lesin del complejo ligamentario psterolateral.
Lesin del ligamento colateral medial.

Trabajando como mdico rural recibimos, en la


urgencia de un centro de salud situado a 100
Km del hospital, una fractura abierta de tobillo
por cada en la montaa, presentando una con
taminacin grave por restos vegetales y tierra.
Nuestra acritud teraputica debe ser:
1.
2.

3.

4.
5.

88.

Lesin nervio mediano.


Lesin nervios radial y mediano.

Nula, evacuacin inmediata solicitando transporte areo.


Antibioterapia, profilaxis antitetnica, retirada
de contaminacin grosera, lavado inicial, cobertura y estabilizacin provisional para traslado inmediato.
Antibioterapia, profilaxis antitetnica, retirada
de contaminacin grosera, sutura de aproximacin para cierre de la herida, inmovilizacin
provisional y traslado inmediato.
Antibioterapia, profilaxis antitetnica y traslado inmediato.
Retirada de contaminacin grosera, lavado
inicial y estabilizacin provisional para traslado inmediato.

Acude a la urgencia una mujer de 53 aos tra


tada dos das antes con una reduccin cerrada y
yeso antebraquial cerrado por una fractura de
Colles en su mueca izquierda. Aqueja dolor
muy intenso y progresivo en mano y mueca, as
.como incapacidad de mover los dedos y falta de
sensibilidad en ellos. El yeso presenta un aspec
to adecuado. Los dedos presentan buen relleno
capilar y una coloracin aceptable, pero estn
muy hinchados, con nula movilidad activa, y su
movilizacin pasiva produce intenso dolor. La
actitud ms correcta ser:
1.

2.

Mantener el yeso, administrar antiinflamatorios y remitir a la paciente a consultas externas.


Mantener el yeso, elevar la mano, ingresar a la
paciente en observacin instndole a mover
activamente los dedos.

Una auxiliar de clnica de 40 aos de edad acu


de a nuestra consulta por dolor intenso en el
codo derecho, irradiado por cara ntero-lateral
de antebrazo, de 4 das de evolucin, que le
dificulta sus actividades cotidianas y laborales.
La movilidad activa del codo es completa, sien
do dolorosa la palpacin del origen de los ms
culos radiales, El dolor empeora con flexin de
codo contra resistencia y con extensin de mu
eca contra resistencia. La radiografa simple es
rigurosamente normal. Nuestra actitud inicial
deber ser:
1.
2.
3.
4.
5.

90.

Serie de 3 infiltraciones con corticoide y anestsico local.


Antiinflamatorios. codera con dispositivo de
presin, aplicacin de calor local.
Resonancia magntica.
Inmovilizacin con yeso braquial.
Indicacin quirrgica inmediata.

Hombre de 29 aos: accidente de trfico.


Ingresa en servicio de Traumatologa con cua
dro de prdida progresiva de fuerza en miem
bros inferiores, los reflejos osteotendinosos
estn abolidos. El estudio con radiografas sim
ples muestran una fractura por compresin de
Lj con desplazamiento del muro posterior y
acuamiento anterior de un 50%, Qu prueba
diagnostica indicara para valorar la ocupacin
del canal raqudeo?:
1.
2.
3.
4.
5.

91.

Abrir el yeso, antiuiflamatorios y diurticos,


estimular la movilidad activa, y si no mejora
rpidamente osteosntesis de la fractura.
Abrir el yeso, administrar aniinflamatorios y
remitir a la paciente a consultas externa.
Abrir el yeso, antiinflamatorios y diurticos,
estimular la movilidad activa, y si no mejora
rpidamente fasciotomia urgente,

Una termografa.
Una densitometra sea.
Una TAC vertebral centrada en regin dorso
lumbar.
Una gammagrafa sea.
Una tomografa cervical.

Tras un accidente de coche llega al Servicio de


Urgencias una paciente de 34 aos inconsciente,
hipotensa, disneica y con fracturas abiertas en
ambas extremidades inferiores. En la valoracin
inicial o revisin primaria rpida y resucitacin,
usted debe realizar el llamado protocolo
ABCDE de la atencin al paciente politraumatizado. Este incluye los siguientes pasos, EXCEP
TO uno, indique este ltimo:
1.
2.
3.
4.

Mantenimiento va area con control de la


columna cervical.
Anlisis del dficit neurogico.
Tratamiento de las fracturas abiertas.
Desvestir completamente a la paciente y preExamen MIR. Versin 0. Enero 2008- PreguntasMIR

venir a hipotermia. 5.
Anlisis del estado circulatorio.
92.

Un paciente poltoxicmano de 28 aos ingres


hace 3 das por fractura subtrocantrea de f
mur izquierdo, fractura de rtula derecha y
fractura diafsaria conminuta de tibia izquierda
que se inmovilizaron provisionalmente en espe
ra de ciruga de osteosntesis. Bruscamente
inicia un cuadro de estupor y obnubilacin
intensos acompaado de disnea y de aparicin
de petequias difusas. Debemos sospechar:

fracaso renal agudo. 5. Mejora el pronstico


del fracaso renal agudo.
96.

Seale la respuesta correcta referida a la Pr


pura de Henoch-Schnlein:
1.

2.
3.

1.
2.
3.
4.
5.

Neumona nosocomial por encarnamiento.


Shock hipovolmico.
Coma exotxico por abuso de sustancias ilegales.
Shock neurognico por dolor.
Sndrome de embolia grasa.

4.
5.

97.
93.

Un hombre de 66 aos, diagnosticado de hiperplasia benigna de prstata, presenta un cuadro


de fiebre con escalofros. El examen de orina
muestra que su pH es 8,5. El sedimento urinario
contiene cristales de estruvita (MgNHjPOj).
Seale la respuesta correcta:
1.
2.
3.
4.
5.

94.

Un paciente de 52 aos con el diagnstico de


ulcus pilrico y vmitos de repeticin de una
semana de duracin acude al Hospital con una
tensin arterial de 100/58 mmHg y la siguiente
analtica: Plasma: Na+ 140 mmol/l, K+ 2.2
mmol/l, Cl" 86 mmol/l, C03H* 42 mmol/l, pH
7.53, pC02 53 mmHg y creatinina 2,9 mg/dl;
Orina Na+ 2 mmol/l, C 21 mmol/l, pH 5. Cul
de los siguientes diagnsticos es cierto?:
1.
2.
3.
4.
5.

95.

Padece una acidosis tubular que le impide


acidificar la orina.
El pH urinario, normal en este paciente, descarta una infeccin.
Cualquier proceso infeccioso bacteriano urinario, eleva el pH.
Debe sospecharse una infeccin urinaria por
grmenes que degradan la urea.
El pH bsico y la presencia de cristales triples,
sugieren infeccin por candidas.

Alcalosis mixta.
Acidosis metablica con vaco amnico
normal
Alcalosis metablica.
Alcaluria paradgica,
Acidosis hipoclormica.

Una mujer de 24 aos acude a urgencias remiti


da por su mdico de Atencin Primaria porque
al realizarle una analtica por astenia, artralgas
febrcula y aparicin de edemas maleolares,
objetiva anemia normoctica normocrmica
(hemoglobina de 9 gs/dl), creatinina srica de 2
mgs/dl, sedimento con microhematuria y pro
teinuria en tira reactiva de 500 mgs/dl. Cul o
cules seran las exploraciones complementarias
que usted solicitara en primer lugar dado el
diagnstico de presuncin?:
1.
2.
3.
4.
5.

98.

3.
4.
5.

1.
2.

1,

3.

2,
3.
4.

Hiperpotasemia.
Imposibilidad de dilisis por falta de acceso
vascular.
Sepsis de punto de partida en acceso vascular.
Hemorragias relacionadas con el uso de heparina.
Enfermedad cardio-vascular.

Seale cul de las siguientes medidas NO es


adecuada para evitar a progresin de la
nefropata diabtica:

Qu es cierto acerca de la oliguria?:


Se define como un volumen urinario inferior a
lOOOml/da.
Es un hallazgo constante en la insuficiencia
renal aguda.
Se asocia a dolor lumbar bilateral.
Hace ms difcil el manejo conservador del

Cuantificacin de proteinuria en orina de 24


horas y aclaramiento de creatinina.
Tomografa axial computarizada renal.
Test de Coombs, haptoglobna, ferrocinetica.
C3, C4, ANA, AntiDNA.
Biopsia renal.

Cul es la causa principal de muerte en pacien


tes con insuficiencia renal crnica, sometidos a
tratamiento con hemodilisis?:
1.
2.

99.

La lesin glomerular, idntica a la de la Neuropata IgA (enfermedad de Berger), se caracteriza por depsitos de IgA en el mesangio
glomerular.
Es una entidad exquisitamente sensible al
tratamiento con corticoides.
Se detectan Ac anticitoplasma de neutrfos
en 80% de pacientes.
Debe sospecharse en pacientes con hemoptisis
e insuficiencia renal aguda.
La presencia de proteinuria masiva es clave en
el diagnstico de esta enfermedad.

4.
5.

Restriccin de protenas en la dieta.


Control glucmico estricto en los diabticos
tipo 1.
Control glucmico estricto en los diabticos
tipo 2,
Incremento de la presin de perfusin glomerular.
Tratamiento con inhibidores de la enzima
conversora de angiotensina, en pacientes con
Examen MIR. Versin 0. Enero 2008- PreguntasMIR

microalbuminuria.

- 15-

100. Cul de estas sustancias es habitualmente utili


zada como marcador tumoral de los tumores de
testculo?:
1.
2.
3.
4.
5.

CEA.
PSA.
AFP.
TPA.
CA 19.9.

101. En un paciente con un traumatismo plvico y


sospecha de rotura uretral por incapacidad de
orinar espontneamente, globo vesical y uretrorragia. Cul de estas maniobras debe evitar
se?:
1.
2.
3.
4.
5.

Sondaje vesical.
Colocacin de cistostoma suprapbica.
Realizacin de uretrografa retrgrada.
Tacto rectal.
Palpacin abdominal.

102. Qu diagnstico debe sospecharse en un hom


bre de 53 aos, fumador, que inicia de manera
insidiosa un sndrome miccional irritativo con
tacto rectal normal, flujo urinario no obstructi
vo, ecografa reno-vsico-prosttica normal,
sedimento urinario con microhematuria y urocultivo negativo, cistoscopia normal y citologa
urinaria con atipias?:
1.
2.
3.
4.
5.
103.

El carcinoma de clulas renales del adulto suele


asociarse con mayor frecuencia a:
1.
2.
3.
4.
5.

104.

Prostatitis.
Litiasis uretral.
Carcinoma in situ vesical.
Hpemefxoma.
Esquistosomiasis.

Adenocarcinoma de pncreas.
Adenocarcinoma de endometrio.
Insuficiencia renal crnica adquirida y dilisis.
Linfoma.
Quimioterapia previa con ciclofosfamida.

Una maestra jubilada de 74 aos con diabetes


mellitus presenta frecuentes episodios de reten
cin urinaria. No presenta incontinencia al
toser o cambiar de postura. En la exploracin
fsica se detecta una neuropata sensitiva en
extremidades inferiores. El residuo postmiccional es de 400 mL. La maniobra de Valsalva
realizada en bipedestacin no provoca prdida
de orina. El estudio urodinmico muestra un
detrusor acontrctil sin obstruccin al vacia
miento vesical. Los estudios de laboratorio son
normales y el tratamiento farmacolgico ha sido
inefectivo. Cul es la opcin teraputica ms
apropiada?:

Examen MIR. Versin 0. Enero 2008- PreguntasMIR

1.
2.
3.
4.
5.
105.

Ante un paciente con insuficiencia renal crnica


y sospecha de uropata obstructiva en una exploracin
ecogrfica. Qu exploracin es la idnea para evaluar el
nivel de obstruccin y la causa?:
1.
2.
3.
4.
5.

106.

3.
4.
5.

Diabetes Mellitus.
Sndrome de deficiencia andrognica asociada a la
edad.
Hipogonadismo hipogonadotropo.
Sndrome de Cushing.
Hipotroidismo.

Seale cul de las siguientes afirmaciones NO es


correcta en relacin con la aplasia medular:
1.
2.
3.
4.
5.

108.

Renograma isotpico.
Cistoscopia,
Tomografa axial computarizada,
Urografia mediante resonancia magntica.
Urografia intravenosa.

Cul cree usted que es el trastorno endocrino


que provoca mayor prevalencia de disfuncin
erctil?:
1.
2.

107.

Sondajes intermitentes.
Sonda urinaria permanente.
Dilatacin uretral.
Uretrolsis.
Ureterotoma transuretral.

Cursa con pancitopenia.


Se trata con transplante de mdula sea.
Puede responder al suero antitirnoctico (ATG).
Cursa con eritroblastos en sangre perifrica.
Los reticulocitos estn descendidos.

Hombre de 27 aos, con antecedentes de enfer


medad de Crohn y reseccin de leon terminal
hace 3 aos, presenta intensa astenia y palidez
cutneo-mucosa. En la bioqumica destaca LDH
2730 UI/L y bilirrubina 1,7 mg/dL. En el hemograma muestra 2,9 x 109 leucocitos/L con re
cuento diferencial normal, hemoglobina 7,5
g/dL, hematocrito 22%, VCM 135 fl y 105 x 109
plaquetas/L. El recuento de reticulocitos es 1%
y en el frotis de sangre perifrica se observan
abundantes neutrfilos hipersegmentados.
Cul sera su juicio diagnstico ms probable y
su actitud teraputica inicial?:
1.
2.
3.
4.
5.

Anemia de probable origen inflamatorio. Transfundir


concentrado de hemates.
Anemia megaloblstca por dficit de vitamina B12.
Iniciar vitamina B12 parenteral y cido flico oral
Anemia hemoltica. Iniciar corticoides.
Sndrome mielodisplsico. Transfundir concentrado
de hemates.
Anemia megaloblstca por dficit de cido flico.
Iniciar cido flico oral.

Examen MIR. Versin 0. Enero 2008- PreguntasMIR

-16109.

Una chica de 27 aos, asintomtica, le consulta


porque en una revisin de la empresa le han
detectado alteraciones analticas. Aporta los
siguientes resultados: hemates 4.9 x 10IZ/L,
hemoglobina 9 g/dl, VCM 75 fl, leucocitos 6.2 x
109/L, plaquetas 220 x 109/L, bilirrubina total
12 jimol/1 (normal: 5 - 17), LDH 2.8 [ikat/l
(normal: 1.7 - 3.2), sideremia 20 jj.mol/1 (nor
mal: 9 - 27) y ferritina 180 U/I (normal: 10 200). Cul es el diagnstico ms probable?:
1.
2.
3.
4.
5.

110.

Ante los siguientes hallazgos analticos:


Hemoglobina 8,5 gr/dL, VCM 85 fl, Bilirrubina
normal, hierro srico 10 pg/dL, capacidad de
fijacin total de hierro 200 U/dL, ndice de
saturacin de !a transferrina 15% y ferritina
150 |ig/dL. Qu tipo de anemia pensara que
tiene el paciente?:
1.
2.
3.
4.
5.

111.

Ciclosporina.
Prednisona.
Hidrocortisona.
Esplenectoma.
Inmunoglobulinas intravenosas.

En una paciente de 20 aos de edad con adenopatas laterocervicales dolorosas tras la ingesta
de alcohol, fiebre y prurito intenso generalizado
el diagnstico ms probable sera:
1.
, 2.
3.
4.
5.

113.

Anemia por dficit de vitamina B12.


Anemia inflamatoria (de proceso crnico).
Anemia ferropnica.
Anemia por dficit de cido flico.
Anemia hemoltica aguda.

Cul es el tratamiento de primera lnea de un


paciente con cifras de plaquetas inferiores a 10
x 109/L, ditesis bemorrgica y un aspirado de
mdula sea con abundantes megacariocitos?:
1.
2.
3.
4.
5.

112.

Anemia ferropnica.
Anemia hemoltica.
Anemia sideroblstica.
Mieiofibross.
|3 talasemia.

Mononucleosis infecciosa.
Toxoplasmosis.
Enfermedad de Hodgkin.
Linfoma difuso de clulas grandes.
Tuberculosis ganglionar.

Mujer de 30 aos que consulta porque en una


revisin de empresa le han detectado un hemograma normal, con leucocitos 35 x 109/L (60%
segmentados, 12% cayados, 16% mielocitos,
4% meta mielocitos, 7% linfocitos, 1% monocitos, hemoglobina 127 g/L, VCM 89 fL, HCM 28
pg, reticulocitos 46 x 109/L y plaquetas 389 x
109/L. La morfologa eritrocitaria es normal. En
el resto de anlisis destaca un cido rico de 8
Examen MIR. Versin 0. Enero 2008- PreguntasMIR

mg/dL y una LDH de 650 UI/L. Su estado general est


conservado, sin ningn otro sntoma que una ligera
sensacin de astenia desde hace 2-3 meses. No ha perdido
peso, no tiene fiebre ni ningn sntoma de infeccin ni de
dolor. La exploracin fsica muestra la presencia de una
esplenomegalia de 1-2 traveses de dedo por debajo del
reborde costal como nico hallazgo. La paciente no es
fumadora y no tiene antecedentes de inters excepto que
hace seis meses se le practic otro hemograma que mostr
una cifra leucocitaria de 14 x 109/L, que se atribuy a una
infeccin respiratoria que resolvi sin problemas. Cul es
el diagnstico ms probable en ste caso?:
1.
2.
3.
4.
5.
114.

Un paciente VIH positivo presenta una gran


masa abdominal, adenopatas a nivel supra e
infradiafragmtico, sudoracin nocturna y
fiebre. La biopsia ganglionar muestra una proli
feracin difusa de clulas de mediano tamao
no hendidas, con un citoplasma intensamente
basfilo con vacuolas, CD19 y CD20 positivas, y
reordenamiento del protooncogn C-MYC. De
entre las siguientes entidades cul es el diagns
tico ms probable:
1.
2.
3.
4.
5.

115.

Linfoma folicular.
Enfermedad de Hodgkin.
Linfoma de Burkitt.
Linfoma de clulas del manto.
Linfoma T hepatoesplnico.

Hombre de 65 aos diagnosticado de mieloma


mltiple IgG Kappa con un nivel de paraproteina de 2.400 mg/dl, sin proteinuria de Bence
Jones, anemia, insuficiencia renal, hipercalcemia ni lesiones seas significativas. La actitud
teraputica inicial debe ser:
1.
2.
3.
4.
5.

116.

Leucocitosis reactiva.
Leucemia mieloide crnica (LMC).
Mielofibrosis en etapa incipiente.
Leucemia mielomonoctica crnica (LMMC).
Sndrome mielodsplstico.

Tratamiento con esquema Meifaln y Prednisona.


Esquema Meifaln, Prednisona y Talidomida.
Autotrasplante de progenitores hematopoyti-cos.
Poliquimioterapia tipo VAD.
No tratamiento inicial, seguimiento e iniciar
tratamiento ante datos de progresin.

Cul de las siguientes afirmaciones sobre el


tratamiento de la trombosis venosa profunda
(TVP) es FALSA?:
1. En pacientes con TVP confirmada objetivamente,
recomendamos tratamiento agudo con heparna de
bajo peso subcutnea o heparna no fraccionada.

Examen MIR. Versin 0. Enero 2008- PreguntasMIR

2.

3.

4.

5.

117.

Hombre de 76 aos con antecedentes de hiper


tensin arterial en tratamiento con enalapril
que consulta por presentar de forma brusca
disartria y hemiparesia derecha que desaparece
en doce horas. Eco~Doppler de troncos supraarticos: ateromatosis carotdea bilateral
con estenosis del 55% en la cartida izquierda,
y del 30% en la derecha* Electrocardiograma:
fbrilacin auricular con respuesta ventricular a
90 Ipm. TAC craneal: normal. Para la preven
cin de nuevos episodios su tratamiento inicial
sera:
1.
2.
3.
4.
5.

118.

5.

2.
3.
4.
5.
120.

121.

2.
3.
4.
5.

Contaminacin bacteriana de la sangre.


Sepsis secundaria al ulcus.
Reaccin febril secundaria a la transfusin.
Reaccin febril por el plasma que contamina
los hemates.
Reaccin transfisional hemoltica.

Acude a nuestra consulta un paciente de 30


aos con fiebre, cefalea y "trancazo" desde hace
48 horas. En la exploracin, adems de fiebre
de 38,5 C, se observa un exantema raaeulopapuloso generalizado que afecta a palmas y plan
tas y una pequea lesin costrosa en una pierna.
Cul de los siguientes es el diagnstico ms
probable?:
1. Fiebre botonosa.

Para su diagnstico es til un frotis y una gota


gruesa.
Podra haberse evitado con una vacunacin
correcta.
Es una enfermedad potencialmente mortal.
Las medidas de barrera son fundamentales en la
prevencin.
La doxicclina ha demostrado eficacia en su
profilaxis.

Hombre de 47 aos, natural de Brasil, residente


en Espaa desde hace 10 aos, acude a su con
sulta por presentar disfagia y estreimiento
crnico. Entre sus antecedentes destaca, ser
portador de marcapasos y una miocardiopata
dilatada. En el trnsito gastroesofgico se obje
tiva un megaesfago, as como megacolon en el
enema opaco. La prueba diagnstica ms ren
table para el diagnstico sera:
1.
2.
3.
4.
5.

124.

Gnero Staphyococcns.
Enterobactericeas.
Bacteroides sp.
Pseudomona aeruginosa.
Streptococcus sp.

Un hombre de 43 aos, ingeniero de profesin,


ha permanecido durante 2 meses en la RD del
Congo. A los 8 das de su regreso, comienza con
un cuadro de fiebre, escalofros, cefalea, mialgias, nauseas y dolor abdominal. A la explora
cin: esplenomegalia y discreta hepatomegalia.
En la analtica sangunea destaca: Hb 9,8 gr/dl;
Leucocitos 3465 mm3, plaquetas 97.000, AST
121, ALT 119 y Na 129. Es FALSO sobre esta
patologa que:
1.

123.

Arbovirus-dengue.
Rickettsias-tifus.
Protozoos-esquistosomasis.
Hongos-histoplasmosis.
Bacterias-lepra.

Los grmenes que con mayor frecuencia infec


tan las heridas quirrgicas limpias son:
1.
2.
3.
4.
5.

122.

Les secundaria.
Tifus murino.
Mononucleosis infecciosa.
Sndrome de Stevens-Johnson.

Cul de las siguientes relaciones entre mi


croorganismo y enfermedad es INCORRECTA?:
1.
2.
3.
4.
5.

Acenocumaro, ajustndolo a la dosis que


consiga un ENR de 2.5.
cido acetil saliclico: 300 mg al da.
Clopidogrel: 75 mg al da.
Angioplastia transluminal percutnea carotdea
izquierda.
Endarterectoma carotdea izquierda.

Paciente de 36 aos que, debido a una hematemesis masiva por un ulcus recibe una transfu
sin con 2U de concentrado de hemates. A los
5-10 m. de iniciarse la transfusin comienza con
fiebre, escalofros, hipotensin, dolor en regin
lumbar y oliguria. Cul sera el diagnstico
ms probable?:
1.
2.
3.
4.

119.

Para pacientes con elevada sospecha clnica de


trombosis, se recomienda tratamiento anticoagulante mientras se espera el resultado de las
pruebas diagnsticas.
El tratamiento con heparina se dice debe controlarse realizando TPT (tiempo parcial de
tromboplastina).
Se recomienda iniciar la administracin de
anticoagulantes orales junto con heparina en el
primer da del tratamiento, e interrumpir la
administracin de heparina cuando el INR sea
estable y superior a 2.
En los pacientes tratados con heparina de bajo
peso es necesario hacer mediciones sistemticas
de la actividad antiXa con el fin de ajustar la
dosis.

Panendoscopia oral y colonoscopa para toma


de biopsias.
Aspirado de mdula sea para Jeishmania sp.
Serologa para T. cruzi.
Coprocultivo para enteropatgenos.
PCR para malaria.

Sealar la respuesta correcta en relacin con la


osteomielitis vertebral:
Examen MIR. Versin 0. Enero 2008- PreguntasMIR

1.

2.
3.
4.
5.

125.

4.
5.

128.

Infeccin por Proteits Mirabilis.


Genero masculino
Presencia de ms de 105 unidades formadoras
de colonias por rnicrolitro de cualquier bacteria.
Infecciones por Stafiococcus Saprophyticus.
Infeccin por Escherichia Coli.

2.
3.
4.
5.

Eritromicina IV ms Cefuroxima IV, hospitalizado,


Amoxicilina-clavulnico IV ms Gentamicina
IV, hospitalizado.
Ceftriaxona IV ms Caritromicina VO, hospitalizado.
Telitromicna IV. hospitalizado.
Levofloxacio durante 7 das, ambulante,

En relacin al tratamiento de la Sfilis. Cul de


las siguientes es la respuesta correcta?:
1.
2.
3.
4.
5.

129.

Un paciente de 80 aos con antecedentes de


insuficiencia renal crnica y diabetes tipo 2,
presenta desde hace 3 das tos con expectora
cin purulenta, fiebre alta, dificultad respirato
ria y dolor costal derecho. En la exploracin
presenta una presin arterial de 120/60 mmHg,
frecuencia cardiaca de 80 LPM, y respiratoria
de 20 RPM. La temperatura es de 38C; se en
cuentra algo confuso y parcialmente desorien
tado. Presenta crepitantes localizados en la base
pulmonar derecha y la Rx de trax muestra un
infiltrado de pequeo tamao en esa localizacin. Cul de las siguientes es la opcin de
tratamiento antibitico ms adecuada?:
1.

127.

monitorizar los niveles plasmticos de los


frmacos. 5. La utilizacin de procedimientos de
administracin directamente observada es
recomendable en determinados pacientes.

Las pautas cortas (3 das) de tratamiento anti


bitico de la infeccin urinaria, NO se aconse
jan en ei caso de:
1.
2.
3.

126.

La forma ms comn de afectacin vertebral


es la diseminacin por contigidad de un foco
paramenngeo.
El agente causal ms frecuente es Escherichia
Coli.
La resonancia magntica es el procedimiento
diagnstico de eleccin.
La evolucin slo con tratamiento mdico es
generalmente desfavorable.
La duracin recomendable del tratamiento
antibitico es de 4 semanas.

Un funcionario de prisiones previamente sano,


no fumador, de 35 aos, con tos y expectoracin
mucoide desde hace 3 meses, acude a consulta
con una radiografa reciente que muestra un
infiltrado cavitado en lbulo superior derecho.
Est eupneico, no tiene fiebre ni dolor alguno.
El examen fsico es normal. Lo ms adecuado
ser ahora:
1.

2.

3.

4.
5.

130.

1.

1.

3.

2.

3.

4.

2.

4.
5.

131.

Recoger al menos 3 muestras de esputo matutino para baciloscopia y cultivo, solicitar Mantoux y anlisis de sangre, incluyendo transaminasas, y serologa de VIH, y aislarle en domicilio hasta tener los resultados.
Avisar a Epidemiologa de la Comunidad
Autnoma para que inicie el protocolo de estudio y tratamiento de los contactos.
Remitirle a Urgencias del Hospital para estudio y tratamiento antituberculoso, durante 2-3
semanas, o hasta que la baciloscopia sea negativa.
Comenzar tratamiento antituberculoso mientras esperamos las bacloscopas de esputo.
Dado que es una persona de alto riesgo, debe
estar vacunado de la tuberculosis y por ello el
Mantoux no es valorable.

Seale la correcta en relacin con la circunci


sin y la prevencin de la transmisin de la
infeccin por VIH:

Entre los principios bsicos del tratamiento de


la tuberculosis NO se encuentra uno de los si
guientes:
La utilizacin de ms de un frmaco al que el
microorganismo sea sensible.
La estrategia de induccin con un nmero
elevado de frmacos durante dos meses, seguida de un tratamiento simplificado de duracin prolongada.
En la actualidad el tratamiento de inicio recomendado est compuesto por soniacida, rifampicina, etambutol y pirazinamida.
En casos de fracaso teraputico se recomienda

No hay ningn tratamiento efectivo.


La Penicilina es el tratamiento de eleccin en
cualquier estadio.
La Penicilina es el tratamiento de eleccin
slo en la Neurosfilis.
La Penicilina es el tratamiento de eleccin slo
en la Sfilis no complicada.
El tratamiento de eleccin son las Tetraciclinas.

En frica se ha mostrado eficaz en la prevencin de la transmisin de la mujer al hombre.


Reduce la transmisin tras relaciones homosexuales entre hombres.
Incrementa la transmisin del VIH del hombre
a la mujer.
El balance coste-beneficio es desfavorable en
pases subdesarrollados.
Se recomienda generalizar el procedimiento
para reducir las tasas de transmisin en todo el
mundo.

Cul sera el tratamiento quirrgico de elec-

Examen MIR. Versin 0. Enero 2008- PreguntasMIR

cin para una lesin cutnea en regin deltoidea


con Anatoma Patolgica de Melanoma Maligno
Clarck IVy Breslow de 1,92 mra:

rosemida, con lo que horas ms tarde la calce-mia


es de 11,5 mg/dl. Cul es el paso teraputico
siguiente ms apropiado?:

1.
2.

1.
2.
3.
4.
5.

3.
4.
5.

Ampliacin de mrgenes de 1 cm.


Ampliacin de mrgenes de 2 cm si la Eeografa axilar es negativa.
Ampliacin de mrgenes de 2 cm y deteccin
del ganglio centinela.
Ampliacin de mrgenes de 2 cm y vaciamiento axilar.
Interfern adyuvante previo al tratamiento
quirrgico.

136.

1.
2.
3.

4.

5.

Sistemticamente diferida al cabo de 5 aos


libre de enfermedad postmastectoma.
El momento {inmediato-diferdo) vendr indicado por el nmero de ganglios positivos.
La quimioterapia postoperatoria es una contraindicacin para realizar la reconstruccin inmediata.
De forma inmediata, pues la reconstruccin de
la mama no interfiere en la historia natural de
la enfermedad.
A partir de los 60 aos es mejor el uso de
prtesis externa.

2.

3.

4.

5.

137.

133. Una mujer de 33 aos consulta por un nodulo


mamario que se diagnostica de carcinoma duc
tal. Su madre a los 50 aos y su hermana a los
40 han tenido cncer de mama. Su ta materna a
los 45 y su abuela a los 55 han tenido cncer de
ovario. Qu estudio solicitara?:
1.
2.
3.
4.
5.

Con respecto a la utilizacin de analgsicos


opioides en enfermos con cncer en situacin
paliativa. Indique Ja respuesta INCORRECTA;
1.

132. Mujer de 60 aos de edad diagnosticada de


carcinoma ductal infiltrante multicntrico de
mama subsidiaria de mastectoma ms vacia
miento axilar y quimioterapia postoperatoria.
Cundo estara indicado realizar la recons
truccin de la mama?:

BCRA1.
Her2.
P53.
Receptor de estrgeno.
Receptor de progesterona.

1.

Una mujer de 70 aos consulta por astenia y


anemia ferropnica que no presentaba en anli
sis del ao anterior. En el estudio se demuestra
un cncer intestinal. En qu lugar estar ms
frecuentemente localizado?:

3.
4.
5.

1.
2.
3.
4.
5.
135.

Duodeno.
Yeyuno.
Ciego.
Sigma.
Recto.

Un paciente con carcinoma de pulmn de clu


las escamosas inoperable desarrolla una hipercalcemia de 14 mg/dl. Se inicia una perfusin
con suero salino y, una vez bien hidratado, fu-

138.

El estreimiento es un efecto secundario casi


constante por lo que se debera utilizar un
laxante para evitar su aparicin.
Su uso continuado requiere aumentar la dosis
administrada para obtener el mismo efecto
analgsico debido a fenmenos d& tolerancia y
sobre todo a la progresin de la enfermedad.
La depresin respiratoria es un efecto secundario frecuente dada la disfuncin orgnica
que presentan estos pacientes.
La morfina no tiene techo teraputico por lo
que no existe una dosis mxima que no pueda
sobrepasarse si lo requiere el paciente.
Se deben pautar a intervalos regulares y no
slo si el paciente presenta dolor.

Un hombre de 84 aos ingresado en una resi


dencia de ancianos est diagnosticado de En
fermedad de Alzheimer moderada. No toma
medicamentos psicotrpicos y duerme bien. En
ocasiones ha presentado ansiedad e ideas deli
rantes de intensidad leve, pero en las ltimas
dos semanas ha pasado cada vez ms tiempo
deambulando por los vestbulos. Varias veces ha
entrado en las habitaciones de otros residentes y
ha burgado en sus pertenencias. En una ocasin
tambin se ha escapado de la Residencia. Cul
de las siguientes estrategias de tratamiento es la
ms apropiada?:

2.
134.

Corticoides.
Nitrato de galio.
Aendronato.
Plicamicina.
Zoledronato intravenoso.

Proporcionarle actividad fsica estructurada y


acompaarlo en sus paseos por el exterior.
Utilizar de forma intermitente un chaleco de
restriccin en una silla durante ei da.
Utilizar restricciones de muecas durante los
episodios de deambulacin ms prolongados.
Prescribir risperidona, 1 mg por va oral, dos
veces al da.
Trasladar al paciente a otro centro geritrico
especializado.

Hombre de 29 aos de edad que es trado a


urgencias de un Centro de Salud por unos veci
nos al habrselo encontrado inconsciente en la
calle. A la exploracin destaca: pulso a 54 lati
dos por minuto, bradpnea con f radas respira
torias cada vez ms frecuentes y prolongadas,
hipotermia, cianosis central y pupilas miticas.
Cul de las siguientes actitudes teraputicas
Examen MIR. Versin 0. Enero 2008- PreguntasMIR

iniciales es la ms correcta?:
1.
2.
3.
4.
5.

139.

No representa un beneficio de la Ciruga sin


ingreso:
1.
2.
3.
4.
5.

140.

Asegurar la va area, oxgeno y suero glucosado 10%.


Asegurar la va area, calentamiento corporal
y observacin.
Asegurar la va area, administrar dopamina
intravenosa y observacin.
Asegurar la va area, administrar naloxona
intravenosa y observacin.
Asegurar la va area, administrar naltrexona y
observacin.

2.
3.
4.
5.
143.

Beneficios psicolgicos, en especial en nios.


Menor infeccin de la herida.
Un menor consumo de analgsicos.
Reduccin de la lista de espera.
Reduccin de los costes sanitarios.

Los opioides son frmacos de uso sistemtico


durante la anestesia. En referencia a dichos
frmacos. Cul de las siguientes afirmaciones
es FALSA?:
1.
2.
3.
4.
5.

Reducen la concentracin alveolar mnima


(CAM) de los anestsicos inhalatorios.
Producen analgesia profunda y rmnima depresin cardiaca.
Pueden producir hipnosis y amnesia.
Pueden producir bradipnea y depresin respiratoria.
Slo son eficaces por va intravenosa,

En qu gen deben buscarse mutaciones en un


paciente joven afecto de un carcinoma medular
de tiroides para determinar si existe un riesgo
familiar a padecer una adenomatosis endocrina
mltiples (MEA)?:
1.
2.
3.
4.
5.

142.

RET.
APC.
MSH2.
c-KIT.
c-MYC.

Las mutaciones del gen GJB1 situado en la


regin cromosmica Xql3.1 son causa de una
neuropata hereditaria denominada CharcotMarie-Tooth tipo CMT1X. Un paciente de 25
aos diagnosticado de CMT1X (caso ndice) y al
que le hemos hallado una mutacin validada
clnicamente acude a la consulta de consejo
gentico con sus dos hijos por el momento clni
camente sanos: un nio de 1 ao y una nia de
3. Cules familiares directos deben ser porta
dores obligados de la enfermedad y en los que
indicaremos pruebas genticas para estudiar su
transmisin a la descendencia?:

Penfigoide ampolloso.
Necrolisis txica epidrmica.
Pnfigo vulgar.
Pnfigo paraneoplsico.
Liquen ampolloso.

Un paciente de 22 aos acude a urgencias en


shock anafilctico, posiblemente despus de
tomar jarabe para la tos. No presenta, antece
dentes personales de inters y a la exploracin
se aprecian lesiones maculosas, marronceas no
descamativas y generalizadas de 1-2 cm. de
dimetro que en zonas de roce estn elevadas,
ms eritematosas y prurignosas. Al ser interro
gado indica que han ido apareciendo y progre
sando en los diez ltimos aos. Qu enferme
dad presenta este paciente que relacione todos
los sntomas?:
1.
2.
3.
4.
5.

146.

Leucoplasia.
Pnfigo.
Enfermedad de Beheet.
Estomatitis aosa recidivante.
Candidiasis oral.

Hombre de 69 aos con antecedentes de hiper


tensin arterial e hiperuricemia en tratamiento
con amlodipino y alopurinol. Hace unas 2 se
manas present un cuadro de lumbalgia por el
que recibi tratamiento con diclofenaco. Acude
a urgencias del hospital por presentar desde
hace 48 horas una erupcin cutnea confluyente
en tronco y extremidades. A la exploracin se
observan lesiones maculares eritematoviolceas
muy extensas (>70% de la superficie cutnea)
sobre las cuales aparecen ampollas y erosiones.
Existe afectacin (lesiones erosivo-costrosas) de
mucosa labial, oral y conjuntiva. El signo de
Nikolsky es positivo. El diagnstico probable es:
1.
2.
3.
4.
5.

145.
141.

Qu lesin de la mucosa oral debe ser conside


rada como precancerosa?:
1.
2.
3.
4.
5.

144.

El hijo de 1 ao.
La nia de 3 aos.
La esposa del caso ndice.
La nia de 3 aos y la madre del caso ndice,

Liquen plano atrfico.


Amiloidosis.
Psoriasis.
Mastoctosis.
Candidiasis mucocutnea crnica,

Cul de estos procesos puede producir simul


tneamente afectacin ocular y cutnea?:
1.
2.
3.
4.
5.

Acn vulgaris,
Lupus eritematoso discoide.
Dermatitis herpetiforme.
Roscea,
Dermatomiositis.

1. Ambos hijos.
Examen MIR. Versin 0. Enero 2008- PreguntasMIR

147.

Con relacin a las manifestaciones clnicas del


liquen plano, todas las respuestas son correctas,
EXCEPTO:
1.
2.
3.
4.
5.

148.

2.
3.
4.
5.

El cuadro clnico expuesto suele ser ms frecuente en personas con hipermetropa.


En esta situacin se espera que la presin
infraocular de ese ojo est elevada.
Se trata de un cuadro ms frecuente en pacientes sin cristalino, (afquicos).
Probablemente este paciente tendr una cmara anterior estrecha.
Es probable que este paciente refiera ver halos
alrededor de las luces.

Hombre de 66 aos que acude a consulta por


dolorimiento y enrojecimiento del ojo derecho
de 2 das de evolucin. La exploracin efectuada
en ese ojo nos revela los siguientes datos: discre
ta disminucin de la agudeza visual, presin
intraocular de 18 mmHg, pupila en miosis, pre
sencia de clulas en humor acuoso, opacidad
cristaliniana moderada y presencia de una sinequia iris-cristalino. Cul sera la actitud inme
diata ms correcta de las que se enumeran a
continuacin?:
1.
2.
3.
4.
5.

150.

5.

151.

Un paciente de 48 aos, acude a urgencias aque


jando un intenso dolor en el ojo izquierdo que le
sobrevino de forma brusca mientras vea televi
sin. A la exploracin se evidencia que ese ojo
izquierdo est enrojecido, con discreto edema
corneal y la pupila se encuentra en midriasis
media, con pobre respuesta a la luz. Ante el
cuadro clnico descrito, qu opcin le parece
MENOS probable de las siguientes propuestas?:
1.

149.

Distribucin simtrica, en zonas flexurales.


Prurito en la mayora de los pacientes.
La variante anular se localiza con preferencia
en el pene.
La variante hipertrfica se localiza preferentemente en el cuero cabelludo.
La afectacin oral aparece en los 2/3 de la
totalidad de los casos.

4.

Ciruga de la catarata de ese ojo, responsable


del cuadro clnico.
Tratamiento tpico (colirios) con midriticos y
corticoides.
Ciruga del gaucoma (trabeculectoma clsica
o esclerectoma profunda no perforante).
Manitol intravenoso asociado a tratamiento
mitco y corticoides tpicos.
Tratamiento con colirios antibiticos tpicos.

5.

3.
4.
5.

153.

Reposo vocal.
Conducta expectante.
Derivacin para bopsia.
Supresin del tabaco.
Braquiterapia.

Cul es el tratamiento de eleccin del absceso


periamigdalino?:
1.
2.
3.
4.
5.

155.
Asociada al HLA-B27Descrita en pacientes VIH.
Muy frecuente en pacientes con lupus eritematoso sstmico.

Es una forma de amigdalitis abscesificada.


Asocia edema de labio con parlisis facial y
tumefaccin parotdea.
Debe ser tratada exclusivamente con antiinflamatorios no esteroideos.
Se produce habitualmente en pacientes inmunodeprnidos, siendo su etiologa mictica.
Es una forma grave de infeccin que afecta al
suelo de la boca y la regin submandibuiar.

Ante un paciente fumador de 45 aos que pre


senta disfona y mediante exploracin se obser
va una lesin vegetante rugosa en el borde libre
de la cuerda vocal derecha cul es la conducta
a seguir?:
1.
2.
3.
4.
5.

154.

Oclusin de la arteria central de la retina.


Obstruccin de la vena central de la retina.
Retinopata diabtica proliferativa.
Retinopata hipertensiva grado IV de KeithWegener.
Hemorragia vitrea.

La angina de Ludwig:
1.
2.

La Uvetis por reconstitucin inmune es una


entidad:
1.
2.
3.

Mujer de 65 aos, hipertensa y diagnosticada


de diabetes no insulino-dependiente, con buen
control metablico, acude a consulta por prdi
da importante de agudeza visual en su ojo dere
cho de 4 das de evolucin. En la exploracin del
fondo de ojo destaca la presencia de venas dila
tadas y tortuosas, hemorragias en llama, edema
retiniano difuso y algunas manchas algodono
sas; no se observan alteraciones significativas en
cabeza de nervio ptico (papila). Con los datos
indicados, seale el diagnstico ms probable:
1.
2.
3.
4.

152.

Que ocurre en algunos pacientes con sfilis


ocular tras la administracin de penicilina.
Que ocurre en algunos pacientes con sarcodosis tras abandonar el tratamiento con corticoides.

Amoxicilina clavulnico.
Corticoesteroides.
Clindamicina.
Eritromicina.
Quirrgico.

El principal germen responsable de la otitis


externa difusa es:
1. Pseudom ono A erugin osa. 1.
Proteas.
Examen MIR. Versin 0. Enero 2008- PreguntasMIR

3.
4.
5.
156.

Can dida A Ibicans.


Hem oph Hits Influenz ae.
Siaphylococus Aureits.

160.

La hipoacusia caracterstica de la enfermedad


de Menire es:

Sealar cul de las siguientes afirmaciones es


FALSA en relacin a los efectos adversos de las
sales de litio en el tratamiento del trastorno
bipolar:
1.

1.
2.
3.
4.
5.
157.

Uno de los siguientes sntomas caractersticos


del sndrome esquizofrnico no puede conside
rarse un sntoma negativo:
1.
2.
3.
4.
5.

158.

2.
3.
4.
5.
161.

Anhedonia.
AJogia.
Frialdad emocional.
Paralogia.
Abulia.

Cul de los siguientes enunciados respecto a la


depresin mayor o episodio depresivo mayor es
INCORRECTO?:
1.

2.

3.

4.

5.

159.

Hipoacusia de transmisin.
Hipoacusia neurosensorial sin reclutamiento.
Hipoacusia neurosensorial luctuante.
Hipoacusia mixta, de transmisin y neurosensorial.
Hipoacusia neurosensorial retrolaberntica.

A lo largo de la vida, aproximadamente un


15% de la poblacin sufre un episodio depresivo mayor.
Cerca de un 8% de los pacientes que atiende
un mdico de Atencin Primara podra recibir
un diagnstico de depresin mayor.
Hasta un 15% de los casos de depresin mayor
son secundarios a una enfermedad mdica somtica o al abuso de substancias.
Cuando se sospecha que un paciente sufre un
episodio depresivo mayor, debe evitarse hacer
preguntas directas al paciente sobre ideas o
proyectos suicidas, porque tal pregunta aumenta el riesgo de suicidio un 35%.
Cerca del 15% de los pacientes con depresin
mayor que no reciben tratamiento se suicidan.

La diferencia fundamental entre "rasgo de per


sonalidad" y "trastorno de personalidad" es la
siguiente:
1.
2.

3.

4.
5.

No existen diferencias entre ambos conceptos.


Se utilizan de modo equivalente.
El rasgo de personalidad es un patrn persistente de funcionamiento cognitivo, afectivo y
relacional inflexible, desadaptativo y causa
malestar.
El trastorno de la personalidad es un patrn
persistente de funcionamiento cognitivo, afectivo y relacional inflexible, desadaptativo y
causa malestar.
El rasgo de personalidad es por definicin
patolgico y como tal debe de tratarse.
El rasgo de personalidad conduce inevitablemente hacia un trastorno de la personalidad.

Mujer de 47 aos sin antecedentes psiquitricos


que es hospitalizada para estudio de metrorragias y a los 5 das de su ingreso es informada del
diagnstico de neoplasia uterina con metstasis.
Se consulta a Psiquiatra porque 24 horas des
pus muestra tristeza y llanto frecuente, refiere
ideas de muerte y presenta insomnio. En la
evaluacin psiquitrica no se recoge ningn
antecedente psiquitrico y la exploracin detec
ta elevada ansiedad y desesperanza en relacin
con las consecuencias de su enfermedad neoplsica. El diagnstico ms probable es:
1.
2.
3.
4.
5.

162.

Trastorno narcisista de la personalidad.


Trastorno paranoide de la personalidad.
Trastorno antisocial de la personalidad.
Trastorno lmite de la personalidad.
Trastorno obsesivo de la personalidad.

La caracterstica ms especfica del sndrome


esquizofrnico es:
1.
2.
3.
4.
5.

164.

Trastorno esquizoafectivo.
Distimia.
Trastorno de ansiedad generalizada.
Trastorno por estrs postraumtico.
Reaccin normal frente al estrs.

Sealar el trastorno de la personalidad en el


que es ms frecuente la presencia de autolesiones:
1.
2.
3.
4.
5.

163.

El litio puede ocasionar alteraciones de la


funcin renal.
El litio puede causar alteraciones de la electrofisiologa cardiaca.
El litio puede provocar exacerbacin de la
psoriasis.
El litio puede provocar insuficiencia respiratoria.
El litio puede producir hpotirodismo.

La existencia de trastornos de la percepcin.


La incapacidad para distinguir entre la realidad
interior y la exterior.
El marcado simbolismo que expresa su conducta.
La hipersensibilidad.
La incongruencia del pensamiento.

Un seor de 60 aos, empleado de comercio,


acude a urgencias acompaado de su familia;
stos cuentan que el da anterior, de forma bas
tante brusca, comenz a decir cosas raras, a no
responder a lo que le preguntaban y a mostrar
se confuso incluso respecto de su propio nom
bre. Efectivamente en la exploracin parece no
Examen MIR. Versin 0. Enero 2008- PreguntasMIR

entender lo que se le pregunta, no recuerda nada de


lo que le ha pasado y no sabe ni el da, ni el lugar
en el que est. El diagnstico sera:

Cul de los siguientes diagnsticos corresponde a


este cuadro?:
1.

1.
2.
3.
4.
5.

Psicosis breve.
Delirium.
Alzheimer.
Esquizofrenia.
Trastorno de ansiedad.

2.
3.
4.
5.

165.

Un paciente bebedor habitual de alcohol es


llevado a urgencias con un cuadro de desorien
tacin, dificultad para recordar hechos de los
das anteriores, errores en los razonamientos,
marcha torpe y una desviacin en los ojos di
vergente que antes no tena. Segn parece, ha
estado bebieodo alcohol hasta unas horas antes
de llevarlo a urgencias. Probablemente se trate
de un caso de:
1.
2.
3.
4.
5.

169.

Sndrome de abstinencia complicado.


Enfermedad de Korsakoff.
Encefalopata de Wernicke.
Alucinosis alcohlica.
Intoxicacin etlica aguda.

Mujer de 60 aos. En la revisin rutinaria se


detecta una masa mvil en pelvis. El estudio
ecogrfico pone de manifiesto, como nico
hallazgo, una lesin qustica de 70 mm. con
proliferaciones papilares internas y dependiente
de ovario derecho. El marcador tumoral Cal25
es de 70 U/mL (normal < 35 U/ml). Cul es la
conducta diagnstico/teraputica adecuada?:
1.
2.

3.
166.

En cul de los siguientes casos la terapia electroconvulsivante es un tratamiento de primera


eleccin?:

4.
5.

1.
2.
3.
4.
5.
167.

170.

1.
2.

1.

4.
5.

3.
4.
5.

Historia clnica familiar pormenorizada ya que


io ms importante son los antecedentes familiares.
Realizacin de una Resonancia Nuclear magntica de la hipfisis para descartar un adenoma.
Estudio ecogrfico ginecolgico con tecnologa tridimensional y biopsia.
Valoracin clnica, estudio hormonal (andrgenos, FSH y LH) y ecografa ginecolgica.
Un escner para estudiar de forma resolutiva
los ovarios y estudio hormonal completo.

Una mujer de 18 aos consulta por amenorrea


primaria. En la exploracin se observa un feno
tipo femenino normal pero con ausencia de
desarrollo de los caracteres sexuales secunda
rios. Los genitales externos son femeninos de
aspecto infantil y los genitales internos femeni
nos e hipoplsicos. La talla es normal. El cariotipo es 46 XX. Los niveies plasmticos de gonadotropinas (FSH y LH) estn muy elevados,

3.

171.

La insuflacin de las trompas.


La inseminacin artificial con semen del marido.
La inseminacin artificial con semen de donante.
Inducir la ovulacin con gonadotropinas.
La fecundacin in vitro.

Cul de las siguientes situaciones clnicas es


una contraindicacin absoluta para anticoncep
cin hormonal?:
1.
2.
3.
4.
5.

172.

Control evolutivo mediante seracin ecogrfica y del nivel del marcador.


Completar el estudio mediante marcadores de
estirpe germinal (alfa-fetoprotena/gonadotropina corinica).
Estudio citolgico de la lesin mediante puncin aspiracin con aguja fina.
Tratamiento quirrgico: anexectoma y estudio
intraoperatorio de la lesin.
Tratamiento quirrgico: quistectoma y estudio
intraoperatorio de la lesin.

Cul es el tratamiento correcto para una pare


ja estril por una obstruccin bilateral de las
trompas?:

El sndrome de ovario poliqustico es una en


fermedad bien conocida de la mujer en edad
frtil y su diagnstico se basa en los datos que
aporta:

2.

168.

Esquizofrenia paranode.
Psicosis reactiva.
Depresin mayor con sntomas melanclicos.
Trastorno equizotpico de la personalidad.
Trastorno delirante crnico.

Sndrome de insensibilidad a los andrgenos


(feminizacin testicular).
Disgenesia gonadal tipo Sndrome de Turner.
Disgenesia gonadal pura.
Defecto congnito de hormona liberadora de
gonadotropinas (Gn-RH).
Sndrome adren ogenital.

Hipertensin arterial bien controlada.


Historia personal de tromboembolismo previo.
Diabetes.
Mujer de menos de 35 aos y fumadora.
Infeccin urinaria.

Los factores pronsticos condicionan la evolu


cin en el cncer de cuello uterino. Cul de
stos NO sera importante para el estadiaje
clnico del tumor?:
1.
2.
3.
4.

Tamao tumoral.
Afectacin ganglionar.
Profundidad de invasin tumoral.
Permeabilidad vascular.
Examen MIR. Versin 0. Enero 2008- PreguntasMIR

5, Vaginitis de repeticin.
178.
173.

A una gestante a trmino que ingresa en trabajo


de parto se le detectan unas pequeas vesculas
vuivares de herpes simple recidivante. Aos
antes de la gestacin tuvo una primoinfeccn
de herpes genital y varios brotes de herpes reci
divante. Cul es la conducta a seguir?:

Seala cul de las siguientes afirmaciones sobre


los genes BRCA1 y BRCA2 implicados en el
cncer de mama hereditario es FALSA:
1.

2.
1.
2.
3.

4.
5.

174.

Cul de los siguientes factores NO es un factor


de riesgo para desprendimiento prematuro de
placenta normalmente inserta?:
1.
2.
3.
4.
5.

175.

4.
5.

4.

5.

179.

Hipertensin arterial crnica.


Nuliparidad,
Tabaquismo.
Hipofibrinogenemia congnita.
Presencia del anticoagulante lpco en sangre.

Definir la causa de un sangrado vaginal.


Confirmar la edad gestacional.
Valorar una sospecha de enfermedad trofoblstica gestacional.
Diagnosticar un embarazo en mujer con amenorrea.
Evaluar un dolor plvico agudo.

Amenaza de aborto.
Aborto diferido.
Mola hidatdica.
Amenaza de aborto en un tero con miomas.
Aborto incompleto.

Paciente de 51 aos que en mamografta de cri


bado se detecta nodulo espiculado de 8 mm. en
cuadrante superoexterno de la mama izquierda.
Biopsia con agua gruesa: carcinoma ductal
infiltrante G IL Receptores hormonales positi
vos. Ki 67 10%, Her 2-neu negativo. Explora
cin clnica: nodulo no palpable, axila libre,
Ecografa axilar: No ganglios sospechosos.
Cul es el tratamiento primario de eleccin?:
1.
2.
3.
4.
5.

180.

181.

Fibrosarcoma del cornete nasal.


Atresia de coanas unilateral.
Rinitis crnica por Rinovirus.
Cuerpo extrao ntranasal.
Granulomatosis crnica nasofarngea.

En relacin con el crecimiento y desarrollo del


nio en su primer ao de vida, todas las afirma
ciones siguientes son ciertas, EXCEPTO:
1.

Cul de las siguientes serologas es ms reco


mendable hacer a todas las embarazadas?:

2.

1.
2.
3.
4.
5.

3.

La del citomegalovirus.
La del parvovirus.
La de la hepatitis C.
La del SIDA.
La de la Iisteriosis.

Hormonoterapia primaria.
Quimioterapia neoadyuvante.
Mastectoma y Knfadenectoma axilar.
Tumorectoma y ganglio centinela.
Trastuzumab.

Paciente de 4 aos que acude a urgencias con


una historia de secrecin nasal purulenta y de
mal olor unilateral desde hace 4 das. El dia
gnstico ms frecuente es:
1.
2.
3.
4.
5.

En una mujer embarazada de 11 semanas que


lleva tres das sangrando por genitales, con
muchas nuseas, tero mayor que su amenorrea
y unos valores de beta HCG muy elevados, en
cul de las siguientes patologas debe pensarse?:
1.
2.
3.
4.
5.

177.

3.

Cul de las siguientes situaciones NO supone


una indicacin primaria de la ecografa obst
trica durante el primer trimestre de gestacin?:
1.
2.
3.

176.

Hacer una cesrea.


Permitir el parto vaginal y tratar con aciclovir al
recin nacido.
No es necesario una conducta especial ya que
el herpes recidivante no tiene riesgo para el
recin nacido.
Permitir el parto vaginal y aislar al recin
nacido de la madre.
Tratar inmediatamente las lesiones con cido
tricloroactico para inactivar el virus y entonces
permitir el parto vaginal.

El riesgo de padecer cncer de mama en mujeres portadoras de mutaciones en alguno de estos


genes es del 60-85%.
El riesgo de padecer adems cncer de ovario
es mayor en las mujeres portadoras de mutacin en el BRCA1 que en las portadoras de
mutacin en el BRCA2.
Los cnceres de mama asociados a mutaciones
en el BRCA1 son generalmente bien diferenciados en comparacin con los cnceres de
mama espordicos.
Mutaciones en los genes BRCA1 y BRCA2
son muy poco frecuentes en cnceres de mama
espordicos.
La prevalencia de mutaciones del BRCA1 es
mayor que la del BRCA2.

4.

En el primer ao de vida, el nio aproximadamente triplica su peso al crecimiento y duplica


su longitud.
El peso del recin nacido suele disminuir ms
del 10% en la primera semana de vida, por
prdida de lquido extravascular, aunque se recupere posteriormente.
Un nio debe ganar alrededor de 30 gramos/da durante el primer mes de vida.
Los reflejos precoces o arcaicos presentes al
Examen MIR. Versin 0. Enero 2008- PreguntasMIR

182.

nacimiento, desaparecen entre los 3 y 4 meses de


edad. 5. Hacia los 2 meses, suele haber hecho su
aparicin la sonrisa voluntaria o social.

3.

Paciente de 3 meses con infopenia, neumona


por Pneumocysts Carinii junto con muguet de
repeticin e incapacidad para ganar peso. El
diagnstico ms probable es:

5.

1.
2.
3.
4.
5.
183.

186.

Deficiencia de IgA.
Inmunodefciencia combinada grave.
Sndrome variable comn de inmunodefciencia.
Enfermedad granulomatosa crnica.
Deficiencia en molculas de adhesin.

Un nio con anorexia, fatigabilidad e irritable,


adoptando una postura con las extremidades
semiflexionadas y abducidas, que Hora al movi
lizarlo y tiene gingivitis hemorrgica e hiperqueratosis folicular, tendr un dficit de:
1.
2.
3.
4.
5.

184.

4.

Vitamina A.
Triptfano.
Vitamina C.
Seleno.
Vitamina K.

Nia de 2 aos que estando previamente bien


sufre episodio brusco de desconexin del medio
e hipertona, con estridor y sialorrea, durante
aproximadamente 2 minutos, quedando poste
riormente somnolienta durante aproximada
mente 5 minutos. A su llegada al Centro de
Salud se objetiva temperatura axilar de 38,9C.
En relacin con esta nia, cul de las siguientes
afirmaciones es correcta?:
Presenta un proceso benigno pero que casi
siempre recurre.
2. Debe remitirse a un Centro Hospitalario para
realizacin de una prueba de neuroimagen.
3. Si el foco causal de la fiebre es claro y banal y
la recuperacin clnica completa, puede manejarse con antitrmicos y observacin domiciliaria sin realizar ninguna prueba complementaria.
4. Ser necesaria Ja realizacin de un EEG ms
adelante.
5. Presenta un riesgo de desarrollo posterior de
alguna forma de epilepsia mayor que el resto
de la poblacin.

Lactante de 3 meses de edad que acude a la


consulta por presentar desde los 15 das de vida,
regurgitaciones postpandrales en casi todas las
tomas y de forma espordica, vmitos en mayor
cantidad. Heces de caractersticas normales.
Lactancia artificial con frmula de inicio y pre
senta un adecuado desarrollo ponderoestatural,
sin prdida de peso. Cul entre los siguientes,
es el diagnstico ms probable?:
1.
2.
3.
4.
5.

187.

El cuadro clnico neonatal puede ser indistinguible de otras infecciones congnitas.


El recin nacido puede encontrarse asintomtico.
Los recin nacidos infectados pero no enfermos no precisan tratamiento.

Alergia a protena de leche de vaca.


Estenosis pilrica.
Intolerancia a protenas de leche de vaca.
Reflujo gastroesofgico madurativo.
Vlvulo intestinal.

En un nio de 11 aos con asma persistente


moderada, y sensibilizacin a caros de polvo
domstico y plenes de ariznicas, gramneas y
olea, todas las siguientes medidas son recomen
dables EXCEPTO una:
1.
2.
3.
4.

5.

Vacunar anualmente contra la gripe.


Tratar la rinitis y la sinusitis coincidentes.
Ensearle a medir el flujo espiratorio mximo
en su domicilio.
Evitar o reducir la exposicin a los alrgenos a
los que est sensibilizado y a otros irritantes de
la va respiratoria.
Emplear como tratamiento de mantenimiento
beta-agonistas de accin larga en monoterapia.

1.

185.

188.

Una de las siguientes caractersticas clnicas NO


es tpica de la intolerancia a disacridos:
1.
2.
3.
4.
5.

189.

Cuando se diagnostica de fimosis a un lactante,


al comentarles sobre el proceso a sus padres
tenemos que tener en cuenta que:

Se encuentra ante un recin nacido a cuya ma


dre se le descubri una seroconversin a Toxoplasmosis durante la gestacin. Cul de las
siguientes afirmaciones es FALSA?:

1.

1.

4.

2.

El estudio de la placenta puede resultar de


utilidad en el diagnstico.
Debe realizar cuanto antes un estudio serolgco al recin nacido.

Diarrea postprandiai.
Deposiciones de olor cido.
Eritema perianal.
Deposiciones explosivas.
Heces abundantes, brillantes y adherentes
(esteatorreicas).

2.
3.

5.

Hasta los dos aos se puede tratar de un proceso fisiolgico.


Es un hallazgo poco frecuente.
Se realiza circuncisin solo en caso de existir
motivos religiosos.
Las adherencias peneanas y la fimosis se trata
de una misma patologa.
En la actualidad el uso de pomadas de
corticoides evita la circuncisin por motivos
mdicos.
Examen MIR. Versin 0. Enero 2008- PreguntasMIR

190.

1.
2.
3.
4.
5.
191.

C Reactiva (PCR) medida en mmol/L como


variable dependiente con el nmero de articulaciones doorosas o hipersensibles de un paciente
con artritis idioptica (variable independiente). El
modelo ajustado en una muestra de 100 pacientes
es el siguiente: fjy = 0,47 + 0,0&* En el modelo.
Cmo se interpreta el coeficiente de 0,08?:

Entre otras manifestaciones, la talla baja est


presente en mltiples sndromes genticos y
polimalformativos, como los que se exponen a
continuacin, EXCEPTO en uno. Cul de ellos
NO suele cursar con talla baja?:
Sndrome de Turner.
Sndrome de Down.
Sndrome de Silver-Russell.
Sndrome de Seckel
Sndrome de Klinefelter.

L Dado que es superior a 0,05, quiere decir que


no existe asociacin entre las variables dependiente e independiente.
2. Es el cambio obtenido en la media de PCR
como consecuencia de pasar a tener una articulacin adicional inflamada.
3. Es el valor medio de la PCR para los sujetos
con una sola articulacin afecta.
4. Su cuadrado se interpreta como el porcentaje
de varianza de la variable dependiente explicado por el modelo lineal.
5. Es el valor medio basal de PCR para los individuos sanos (sin articulaciones afectas).

Recin nacido prematuro de 3 horas de vida


que presenta cianosis persistente, con hipoxemia, que mejoran escasamente con 02 al 100%.
La radiografa de trax muestra pulmones hi~
perclaros, con corazn en bota. Seale la res
puesta correcta:
1.
2.

3.
4.
5.

El nio presenta una enfermedad de la membrana hialina.


Debe instaurarse una terapia intravenosa con
prostaglandina Ei para mantener permeable el
conducto arterioso.
La ecocardiografa bidmensional no ser
diagnstica en este caso.
El tratamiento de eleccin inicial es la ventilacin mecnica.
El paciente rene todos los criterios de insuficiencia cardiaca neonatal.

195.

1.
2.
3.
4.
5.
196.

192.

Si se realiza el diagnstico de osteocondritis de


la epsis femoral superior o enfermedad de
Legg-Calv-Perthes es verdad que:
1.
2.
3.
4.
5.

193.

1.
2.
3.
4.
5.
194.

La edad de aparicin es en mayores de 10


aos.
Aparece especialmente en el sexo femenino.
La cojera es el sntoma predominante con
mayor o menor dolor.
En la exploracin del nio llama la atencin la
importante afectacin del estado general.
La evolucin de la enfermedad es rpida,
siendo a evolucin ms frecuente de aproximadamente un mes.

En un estudio clnico, a una serie de pacientes se


les trata con un nuevo frmaco para estudiar, si
, en un periodo de tiempo despus de la administracin de dicho frmaco, el nivel de bilirrubina ha
disminuido. Se acepta que la distribucin de la
bilirrubina es normal, para este diseo. Cul es la
prueba estadstica de eleccin?:
T de Student para datos apareados.
T de Student para datos independientes.
Ji-cuadrado.
Mann-Whitney.
Prueba exacta de Fisher.

Supongamos un modelo de regresin lineal sim


ple que relaciona los niveles medios de Protena

En un contraste de hiptesis estadstico, si la


hiptesis nula fuera cierta y se rechazara:

En un ensayo clnico para comparar dos frma


cos, se incluyen 75 pacientes en cada rama, la
variable de inters es tiempo hasta que desapa
recen los sntomas y el perodo de seguimiento
es de 30 das. AI final de ese perodo hay pacien
tes a los que no les han desaparecido los snto
mas. El anlisis de los datos:
1.
2.
3.
4.
5.

197.

Se comete un error de tipo EL


Se toma una decisin correcta.
La potencia aumenta.
Se comete un error de tipo I.
Se toma la decisin ms conservadora.

Se puede hacer con la prueba t de Student.


Se debe hacer con la prueba ji-cuadrado.
Se debe hacer con las tcnicas de anlisis de
supervivencia.
Se debe hacer con la prueba de MarunWithney.
Se puede hacer con la prueba de MannWithney, o con las tcnicas de anlisis de supervivencia.

Un ensayo clnico sobre 3.000 pacientes compa


ra un nuevo frmaco antihipertensivo con pla
cebo (1.500 pacientes por rama de tratamiento),
Para cada grupo, se obtiene un valor medio de
tensin arterial. Para comprobar si existen dife
rencias estadsticamente significativas, se debe
realizar la siguiente prueba:
1.
2.
3.
4.
5.

La X2 de Pearson.
La t de Student.
La F de Snedecor.
La U de Marm-Whitney.
Hazard Ratio.

Examen MIR. Versin 0. Enero 2008- PreguntasMIR

198.

Se va a realizar un estudio para comprobar si la


administracin de determinado antipsictico
podra estar relacionado con un aumento de la
incidencia de conductas suicidas en esquizofr
nicos en los que se inicia el tratamiento de un
brote agudo. Los pacientes ms graves pueden
ser ms proclives a serles indicado este antipsi
ctico y tambin tienen mayor riesgo suicida.
Teniendo en cuenta este posible factor de confu
sin "por indicacin'*. Cul de los siguientes
tipos de estudio minimiza mejor en el diseo la
posibilidad de que se produzca este sesgo de
confusin?:
1.
2.
3.
4.
5.

en este grupo la prevalencia de cardtopatia isqumica sintomtica. 5. Seleccionamos un grupo


de pacientes diagnosticados de infarto agudo de
miocardio y un grupo seleccionado de forma
aleatoria procedente de la misma colectividad, sin
infarto. Comparamos en ambos grupos la
proporcin de pacientes previamente expuestos a
determinado anti-inflamatorio no esteroideo.
201.

Un ensayo clnico aleatorizado.


Un estudio de casos y controles apareado.
Un estudio de cohortes de seguimiento prospectivo,
Un estudio de prevalencia.
Un estudio de casos y controles anidado en
una cohorte.

Si en un estudio de casos y controles se usa la


odds radio para medir la asociacin entre la
exposicin de inters y la caracterstica estudia
da y el error que se comete en la clasificacin de
los sujetos en expuestos o no expuestos es igual
en el grupo de los casos que en el de los contro
les. Cul es el efecto sobre el valor estimado de
la fuerza de la asociacin?:
1.
2.
3.

199.

Respecto a los estudios epidemiolgicos de co


horte es cierto que:
4.
1.
2.
3.

4.

5.

200.

Se utiliza la Odds Ratio como medida fundamental de asociacin.


Se denominan tambin estudios de prevalencia.
El grupo de comparacin se constituye con los
sujetos no expuestos al factor de riesgo en estudio.
Estudian las caractersticas de una muestra de
sujetos en un momento dado, en el que se recogen todos los datos.
Si existen prdidas en el seguimiento de los
sujetos, el estudio no puede analizarse.

5.

202.

1. Seleccionamos un grupo de pacientes diagnosticados de agranulocitosis y un grupo control de


pacientes del mismo hospital con otras enfermedades
hematolgcas. Comparamos a los dos aos en ambos
grupos la proporcin de pacientes que sobreviven. f
2. Seleccionamos un hospital en el que han ocurrido
una serie de casos de legionelosis y como control,
otro hospital sin ningn caso. Comparamos la
temperatura del agua en los depsitos de agua
caliente de cada hospital.
3. Seleccionamos un grupo de diabticos e hipertensos en tratamiento con enalaprlo y otro
grupo con las mismas enfermedades en tratamiento con diurticos. Comparamos en ambos
grupos los valores de protenuria despus de 6
meses de tratamiento.
4. Seleccionamos una muestra de sujetos que han
tenido un resultado positivo en un control de
alcoholemia mientras conducan. Calculamos

Cul de las siguientes afirmaciones es FALSA


sobre las ventajas de los estudios de casos y
controles?:
1.
2.

Cul de los siguientes estudios se corresponde


con el diseo de un estudio de casos y contro
les?:

3.
4.
5.

203.

Se produce un sesgo en el sentido de aumentar


errneamente la fuerza de la asociacin medida.
Se produce un sesgo en el sentido de disminuir
errneamente la fuerza de la asociacin medida.
Se produce un sesgo unas veces aumentando y
otras diminuyendo errneamente la fuerza de
la asociacin medida.
Una estimacin vlida de la fuerza de la asociacin, pero con un aumento de la magnitud
de su intervalo de confianza.
Una estimacin vlida de la fuerza de la asociacin medida.

tiles para abordar enfermedades infrecuentes.


Alternativa eficiente para el estudio de enfermedades con periodos de latenca prolongados.
Facilidad de constituir un grupo adecuado de
comparacin.
Permiten valorar diferentes factores etiolgi-cos
simultneamente.
Un menor plazo de realizacin, unido a un
menor tamao muestral, supone un menor coste.

Los resultados de los distintos estudios observacionales (cohortes, casos y controles) llevados a
cabo sobre el riesgo de infarto agudo de mio
cardio asociado a medicamentos han sido:
- frmaco A: OR(IC95%): 4,0 (2,5-7,2)
- frmaco B: RR(IC95%): 2,1 (0,85-3,9)
- frmaco C: RR(IC95%): 0,7 (0,4-0,85)
- frmaco D: OR(IC95%): 3,0 (2,2-9,1)
Si asumimos que A, B, C y D tienen N misma
eficacia, similar seguridad respecto al resto de
reacciones adversas y similares costes. Cul sera
el tratamiento de eleccin?:

Examen MIR. Versin 0. Enero 2008- PreguntasMIR

1.
2.
3.
4.
5.

Frmaco B,
Frmaco C.
Frmaco A.
Frmaco D.
No puede establecerse pues dependera de la
frecuencia absoluta de la enfermedad (IAM).

3.

4.
204.

Una de la siguientes afirmaciones con respecto a


la aleatorizacin es INCORRECTA:
5.
1.
2.

3.

4.

5.

205.

El proceso de asignacin aleatoria debe estar


documentado en todos sus pasos.
La aleatorizacin permite que la asignacin de
cada paciente incluido en el ensayo clnico sea
impredecible.
En el caso de un ensayo con aleatorizacin
1:1, todos los participantes tienen las mismas
probabilidades de ser asignados a uno u otro
de los grupos en estudio.
La aleatorizacin asegura totalmente la homogeneidad entre los grupos en comparacin ya
que este es su objetivo.
Idealmente el proceso de aleatorizacin debe
estar enmascarado para evitar sesgos en la
asignacin a los grupos en comparacin.

207.

Ensayos clnicos multicntricos.


Ensayos clnicos aleatorizados, doble ciego y
paralelos.
3. Ensayos clnicos post-comercializacin.
4. Ensayos clnicos pragmticos.
5- Diseos de cohorte amplia.

En el diseo y realizacin de un ensayo dioico


debemos tener en cuenta diversos factores a la
hora de determinar los objetivos y variables de
evaluacin que vamos a establecer. Seale la
respuesta correcta:

208.

Debemos establecer tantos objetivos y variables principales de evaluacin como deseemos


evaluar.
2. Si queremos evaluar resultados vlidos y significativos desde el punto de vista clnico, la utilizacin de variables intermedias en un mtodo
ideal y carente de sesgos.
3. La utilizacin de escalas de evaluacin multidimensionales (por ejemplo las de calidad de
vida) tiene la ventaja de ser clnicamente ms
relevante pero puede tener problemas de validacin y ser de difcil interpretacin.
4. A lo largo del desarrollo de un nuevo frmaco
(Fases I a IV) la variable principal de evaluacin de los diferentes ensayos clnicos realizados debe ser la misma para permitir la consis, tencia en la evaluacin de la eficacia del frmaco,
5. El objetivo principal de un ensayo clnico en
fase ni debe ser elegido por su facilidad para
ser medido y as facilitar la deteccin de los
efectos del frmaco.

2.

De las siguientes situaciones clnicas indique en


cul podra estar ms justificado el empleo de
un estudio clnico con control histrico para
evaluar la eficacia:
1.
2.
34.
5.

209.

2.
3.
4.
5.

El denominado "anlisis por protocolo" puede


incurrir en sesgos debidos a las prdidas durante el estudio.
Los estudios con controles histricos tienden a

210.
-28

Hipertensin arterial,
Infeccin de vas respiratorias altas.
Cncer de pncreas en estadio avanzado.
Carcinoma basocelular cutneo.
Dolor postoperatorio agudo despus de extraccin del tercer molar.

Al analizar los resultados de un ensayo clnico,


se concluye que existen diferencias entre los
parmetros evaluados para cada una de las
ramas de tratamiento, con una P de 0,034. Esto
significa:
1.

En relacin con el diseo y realizacin de un


ensayo clnico, seale la respuesta INCORRECTA:
1.

Al contrario que los estudios de eficacia, que se


realizan en condiciones experimentales ideales,
los ensayos clnicos de efectividad pretenden
evaluar el valor de un nuevo tratamiento en
condiciones lo ms parecidas a las que tienen
lugar en la prctica clnica habitual. Este tipo
de ensayos clnicos para la evaluacin de la
efectividad se denominan:
1.
2.

1.

206.

sobrevalorar la eficacia del frmaco experimental.


En el anlisis de un ensayo clnico se debe
hacer el mximo nmero posible de anlisis
intermedios de la variable principal porque as
se aumenta la probabilidad de obtener un resultado positivo en el ensayo.
El ensayo clnico controlado y aleatorizado se
considera el diseo con mayor fortaleza para
determinar la eficacia de una intervencin.
El ensayo clnico cruzado es muy sensible a
las prdidas de sujetos durante la realizacin
del mismo.

Que si se repite el estudio, en un 3,4% de


ocasiones no se encontraran diferencias.
Que en un 3,4% de sujetos de ambas ramas, no
se encontraron diferencias.
Que hay un 3,4% de diferencia entre los tratamientos estudiados.
Que la magnitud de la diferencia entre tratamientos es mayor que si P = 0,05.
Que hay una probabilidad de 3,4% de que la
diferencia obtenida se deba al azar.

Se compara la utilidad de dos frmacos activos,


A y B, en un ensayo clnico cuya variable prin
cipal es la reduccin de la mortalidad asociada
a infarto agudo de miocardio. Los resultados
Examen MIR. Versin 0. Enero 2008- PreguntasMIR

no muestran diferencias estadsticamente significativas en la reduccin de la mortalidad, pero e


anlisis de 10 variables secundarias muestra que en
una de ellas (la reduccin de la estancia media
hospitalaria post-infarto), el frmaco A es superior
al B. Qu actitud tomara ante estos resultados?:

luar nuevas indicaciones del medicamento,


aunque ste ya est en el mercado.
213.

En relacin a los estudios de bioequivalencia,


seale la respuesta INCORRECTA:
1.

Para que se autorice la comercializacin de un


medicamento genrico suele ser necesario realizar un estudio de bioequivalencia en voluntarios sanos.
2. El objetivo de los estudios de bioequivalencia
es demostrar que dos formulaciones del mismo
principio activo tienen la misma eficacia clnica.
3. Las dos formulaciones a comparar deben de
tener la misma composicin cualitativa y cuantitativa en cuanto a principio activo, para que
el producto pueda comercializarse.
4. Se suele emplear un diseo cruzado, de manera que un grupo de sujetos recibe primero la
formulacin de referencj;; . luego la del producto genrico, y el otro grupo la secuencia
inversa.
5. La variable principal de evaluacin se basa en
la cuantificacin plasmtica (o urinaria) del
principio activo, tras la administracin de las
dos formulaciones.

1.

Formular una nueva hiptesis basada en los


resultados observados en las variables secundarias significativas y planificar un nuevo ensayo clnico.
2. Ajustar los resultados de mortalidad por las
diferencias observadas en la estancia media
hospitalaria.
3. Aadir la indicacin de disminucin de la
estancia media hospitalaria en la ficha tcnica
del frmaco A.
4. Realizar un anlisis por subgrupos de pacientes para determinar si la mortalidad est disminuida en alguno de ellos.
5. Agrupar las variables secundaria y primaria en
una nica variable combinada.
211.

Se compara un frmaco activo con placebo para


la prevencin de fracturas osteoporticas en
mujeres postmenopasicas tratadas con corticosteroides. En la interpretacin de este ensayo
clnico, qu significado tiene el nmero necesa
rio para tratar o NNT?:
1.

2.

3.
4.

5.

212.

2.

3.
4.

5.

Las fases son consecutivas en el tiempo pero


pueden superponerse.
Los primeros estudios que se realizan en
humanos para demostrar la eficacia de un medicamento se engloban dentro de la fase I.
Uno de los objetivos de la fase II es establecer
la relacin dosis-respuesta del medicamento.
La utilidad en la prctica clnica se pone de
manifiesto en la fase HI, en la que se incluye a
un nmero mayor de pacientes que en la fase
II.
Los estudios de fase IV se disean para eva-

Qu es un metaanlisis?:
1.

El nmero necesario de pacientes que deben


recibir el comparador no activo para que los
resultados resulten crebles.
El nmero mnimo de pacientes incluidos en el
ensayo clnico que deben concluir el tratamiento para que se mantenga el poder estadstico.
El nmero de pacientes a tratar para que el
frmaco activo resulta coste/eficaz.
El nmero de pacientes que debe incluir el
ensayo clnico para obtener resultados con un
nivel de significacin estadstica predeterminado.
El nmero de pacientes a tratar con el comparador activo para evitar un desenlace adverso.

Una de las siguientes afirmaciones sobre las


cuatro fases del desarrollo clnico de un medi
camento es FALSA, selela:
1.

214.

2.

3.
4.

5.

215.

Cuando se evala la eficiencia de una interven


cin sanitaria. A qu pregunta se est tratando
de responder?:
1.
2.
3.
4.
5.

216.

Un sistema de anlisis estadstico que agrupa


las variables secundaras para aumentar la potencia estadstica de un estudio determinado.
Una revisin sistemtica de la literatura sobre
un determinado tema realizada con criterios
explcitos en fuentes bibliogrficas.
Una metodologa estadstica centrada en el
anlisis econmico de los frmacos.
Un sistema de asignacin aleatoria de pacientes dentro de un ensayo clnico cuyo objetivo
es garantizar la ausencia de sesgos de seleccin.
Un sistema de anlisis para agrupar y resumir
conjuntamente los resultados de diferentes ensayos clnicos sobre una misma materia.

Puede funcionar en condiciones ideales?.


Puede funcionar en la prctica clnica habitual?.
Es adecuada su relacin beneficio-riesgo?.
Compensa es rentable desde el punto de
vista econmico?.
Es adecuada su calidad?.

Cul es el principal inconveniente de las eva


luaciones econmicas realizadas dentre de los
ensayos clnicos?:
1. La necesidad de utilzar suposiciones.
Examen MIR. Versin 0. Enero 2008- PreguntasMIR

2.
3.
4.
5.
217.

La baja validez interna.


La baja validez externa.
La baja calidad de los datos de eficacia.
La falta de rigor cientfico.

5. Disponer de tres subtipos del virus de la gripe


A(H3N2,HTNlyH5Nl).
221.

El resultado de un electrocardiograma de es
fuerzo, para la prediccin de una enfermedad
coronaria, cambia si se varan los milmetros de
depresin del segmento ST que definen el resul
tado anormal. Si se considera como resultado
anormal a partir de los 2 mm de depresin del
segmento ST en lugar de a partir de 0,5 mm.
Seala la respuesta correcta:

Seale la asociacin correcta frmaco antide


presivo - mecanismo de accin:
1.
2.
3.
4.
5.

1.
2.

3.
4.
5.

218.

Cul de los siguientes datos NO forma parte


del vigente conjunto mnimo bsico de datos, al
alta de hospitalizacin, aprobado por el Consejo
Interterrtorial del Ministerio de Sanidad y
Consumo?:
1.
2.
3.
4.
5.

219.

2.
3.

4.
(5.

222.

Cul de los siguientes antiepilpticos adminis


trado en mujeres embarazadas puede producir,
en el recin nacido, una deficiencia de factores
de la coagulacin dependientes de vitamina K?:
1.
2.
3.
4.
5.

223.

Fecha de Nacimiento.
Identificacin del Hospital.
Tipo de Hospital.
Diagnsticos, codificados.
Procedimientos quirrgicos u obsttricos,
codificados.

Las vacunas del papilomavirus frente al carci


noma cervical han demostrado su eficacia
mediante:
1.

220.

La sensibilidad aumenta.
Es necesario representar los falsos negativos
frente a los verdaderos negativos en una curva
ROC.
La especificidad aumenta.
Los falsos positivos aumentan.
Es necesario representar los falsos negativos
frente a los falsos positivos en una curva ROC.

La demostracin de que no aparece cncer


cervical en los vacunados.
La demostracin de que se producen anticuerpos tras la vacunacin.
La demostracin de proteccin frente a la
aparicin de lesiones preneoplsicas (CEN 2/3)
asociadas a los virus incluidos en la vacuna.
La demostracin de la remisin del cncer
cervical en las mujeres vacunadas.
Estudios clnicos no controlados.

2.
3.

4.
5.

1.
2.
3.
4.

Impedir mediante la inmunidad que generan


que aparezca una pandemia.
Precisar solo la administracin de una dosis a
lo largo de la vida.
Ser ms eficaces en ancianos, ya que responden con ttulos de anticuerpos ms elevados.
Variar su composicin antignica cada ao,
como consecuencia de la deriva antignica de
los virus gripales A y B.

225.

Lmezolid.
Vancomicina.
Caspofungna.
Meropenem,
Gentamicina.

Cul de los siguientes criterios que se utilizan


para establecer la relacin de causalidad entre
un frmaco y una supuesta reaccin adversa, es
el nico que resulta imprescindible para al me
nos poder mantener la sospecha de que estamos
ante una reaccin adversa a medicamentos?:
1.

Las vacunas frente a la gripe que se utilizan


actualmente se caracterizan por:

Carbamazepina.
Fenitona.
Etosuximida.
Valproato sdico.
Clonazepam.

Cul de los siguientes frmacos debe de utili


zarse con precaucin en un paciente tratado con
un inhibidor selectivo de recaptacin de serotonina por el riesgo de provocar un cuadro de
hiperactividad serotoninrgica?:
1.
2.
3.
4.
5.

224.

Sertralina - inhibidor de MAO.


Bupropin - inhibidor selectivo de la recaptacin de 5HT.
Iproniacida - inhibidor selectivo de la recaptacn de 5HT.
Citalopram - inhibidor selectivo de la recaptacin de 5HT.
Duloxetina - inhibidor de MAO.

Que la respuesta al frmaco sea conocida


(plausibilidad biolgica).
Mejora de la reaccin al retirar el frmaco.
Secuencia temporal razonable entre la exposicin al frmaco y la aparicin de la reaccin
adversa.
Reaparicin de la reaccin al reintroducir el
frmaco.
Presencia de una explicacin alternativa para
la reaccin (por ejemplo, la propia patologa
de base del paciente).

Seale el frmaco que debe ajustarse su dosis en


el paciente con deterioro de la funcin renal:
1.
2.
3.

Teofilina.
Carbamazepina.
Digoxina,
Examen MIR. Versin 0. Enero 2008- PreguntasMIR

4.
5.
226.

P. aenginosa.
C. albicans.
5". aitrens.
E. coli.
S. epidermidis.

Un nio de 6 meses presenta tos persistente y


fiebre. La exploracin fsica y la radiografa de
trax sugieren una neumona. Cul de los si
guientes microorganismos es menos probable
que sea el agente causal de esta infeccin?:
1.
2.
3.
4.
5.

228.

1.
2.
3.
4.
5.

Qu microorganismo es el principal responsa


ble de las bactericurias asociadas a catter intra vascular?:
1.
2.
3.
4.
5.

227.

Propanolol.
Verapamilo.

4.
5.

4.
5.
232.

233.

Serologa.
Hemocultivo.
Examen microscpico de las heces (T. de
Ziehl-Neelsen modificada).
Coprocultivo.
Reaccin en cadena de Ja pomerasa de una
muestra de sangre.

2.

3.
4.

5.

230.

El VHC es una importante causa de hepatitis


postransftisional,
El VHD es un virus defectivo con un genoma
de RNA y una cpside que contiene el antgeno de superficie del VHB.
El VHD se transmite fundamentalmente por
va fecal-oral.
Los pacientes infectados por el VHC suelen
desarrollar una infeccin crnico-activa que
puede conducir al desarrollo de un hepatocarcinoma.
La determinacin seriada de la carga vrica del
VHC es til para monitorizar la eficacia del
tratamiento administrado.

La organela celular ms implicada en la pato


gnesis de la apoptosis a travs de la va intrn
seca es:

Una biopsia renal muestra con el microscopio


ptico numerosos glomrulos con semilunas. La
inmunofluorescencia presenta un patrn lineal
con la IgG. Cul es el diagnstico?:

Un tumor gstrico protruye en la luz. Est for


mado por clulas epitelioides y fusiformes y con
inmunohistoqumica expresa c-kit (CD117).
Cul es el diagnstico?:
1.
2.
3.
4.
5.

Las siguientes afirmaciones referidas a los virus


de la hepatitis C (VHC) y al virus de la hepatitis
D (VHD), EXCEPTO una, son correctas. Seala
la respuesta INCORRECTA:
1.

Hipertrofia vellosa de la sinovial.


Focos de necrosis caseiforme.
Hiperplasia del lecho que bordea los sinoviocitos.
infiltracin linfoplasmocitaria.
Neo-angiognesis.

. Granulomatosis de Wegener.
2. Sndrome de Goodpasture.
3. Prpura de Schonlein Henoch.
4. Poliarteritis microscpica.
5. Lupus eritematoso sstmico.

234.
229.

La sinovitis de la artritis reumatoide asocia las


alteraciones siguientes, SALVO una. Cul?:
1.
2.
3.

Virus sincitial respiratorio.


Adenovirus.
Virus parainfluenza 1.
Rotavirus.
Virus parainfluenza 3.

La criptosporidiosis, que est causada por Cryptosporidium parvum, cursa como una enteritis
benigna en personas previamente sanas, pero en
personas con inmunosupresin causa una en
fermedad grave. En estas formas graves su
diagnstico convencional se efecta por:
1.
2.
3.

231.

Ncleo.
Mitocondria.
Retculo endoplsmico.
Lisosoma.
Zona interna de la membrana celular.

Cul de los siguientes tumores tiroideos puede


aparecer en el seno de un sndrome de neoplasia
endocrina mltiple?:
1.
2.
3.
4.
5.

235.

Tumor del estroma gastrointestinal (GIST).


Leomiosarcoma.
Adenocarcinoma de tipo intestinal.
Linfoma.
Leiomioma epteJioide.

Carcinoma papilar.
Carcinoma folicular.
Carcinoma de clulas de Hurthle.
Carcinoma medular.
Carcinoma anaplsico.

Un hombre de 30 aos present un tumor intracerebral frontal derecho que no captaba con
traste con los mtodos de imagen. Se le realiz
una lobectoma frontal. Se le aplic radiotera
pia asociada a temozolomida. AI cabo de 8 aos
se produce una recidiva del tumor, que ahora
muestra con los mtodos de imagen captacin
de contraste y reas de necrosis. El estudio his
tolgico estableci el diagnstico de:
1.
2.
3.
4.

Gangliocitoma,
Astroctoma Piloctico.
Glioblastoma.
Meduloblastoma.
Examen MIR. Versin 0. Enero 2008- PreguntasMIR

5.

Cordoma.

4.

236. Las siguientes estructuras desembocan en la


aurcula derecha, EXCEPTO:
1.
2.
3.
4.
5.

Vena cava superior.


Vena cava inferior.
Seno coronario.
Vena pulmonar derecha.
Venas cardacas anteriores.

5.

241.

1.
2.
3.
4.
5.
238.

El esfago tiene capa mucosa, capa muscular y


capa serosa.
El esfago abdominal es ms largo que el
cervical.
El dimetro mximo del esfago normal es de
5 cm.
El esfago torcico pasa por detrs del cayado
artico.
El epitelio esofgico normal es cilindrico.

3.
4.

5.

242.

1.
2.
3.
4.
5.

2.
3.

El sangrado es probable que sea debido a la


seccin de la arteria cubital localizada en el
margen lateral de la mueca.
El sangrado es secundario a la seccin accidental de la arteria humeral.
Posiblemente la seccin de la arteria radial es
la causante del sangrado pulstil.

244.

Terapia sustitutiva con nmunoglobulina intravenosa (IVG).


JL-2 en perfusin intravenosa continua.
Administracin de anti-TNF-alfa.
Administracin de IFN-beta.
Transpiante de progenitores hematopoyticos
(TMO).

En los dficits de la inmunidad humoral (anti


cuerpos). Qu agente de los sealados es ms
probable que cause infeccin oportunista?:
1.
2.
3.
4.
5.

Nervio hipogloso derecho.


Nervio glosofarngeo derecho.
Nervio neumogstrico derecho.
Nervio facial derecho.
Nervio estatoacstico derecho.

Paciente de 18 aos que acude a urgencias por


presentar sangrado pulstil a nivel de mueca
derecha despus de corte accidental con un
cristal. A su llegada a urgencias se coloca man
guito de presin en brazo derecho a una presin
de 120 mmHg con disminucin y desaparicin
del sangrado. Indique cul de las siguientes
afirmaciones es correcta:
1.

2.
3.
4.
5.
243.

La artritis reumatoide y la enfermedad de


Crohn pueden tratarse con anticuerpos monoclonales anti-TNF alfa.
Los pacientes con esclerosis mltiple mejoran
notablemente con la administracin de gamma
interfern.
La plasmafresis es til en el tratamiento del
sndrome de Guillain-Barr.
Los anticuerpos monoclonales anti-CD3 son
muy tiles en el tratamiento del rechazo crnico.
La rapamicina bloquea a la calcineurina unindose a FKBP12 en el citoplasma celular.

Cul de las siguientes estrategias teraputicas


constituye actualmente el tratamiento de elec
cin en las inmunodeficiencias combinadas se
veras (SCID)?:
1.

Mesentrica inferior.
Mesentrica superior.
Esplnica.
Gstrica derecha,
Gstrica izquierda.

239. Indique qu estructura nerviosa ms frecuen


temente puede ser lesionada ante un paciente
con lesin inciso-contusa en regin preauricular
derecha:

240.

2.

Existen anomalas anatmicas, pero habitualmente la arteria gastroepiploica izquierda es


una rama directa de una de las siguientes arte
rias:
1.
2.
3.
4.
5.

Sobre la inmunoterapia es correcto que:


1.

237. En la ciruga del cncer de esfago es importan


te conocer bien la anatoma esofgica. Cul de
las siguientes afirmaciones es correcta?:

La vena ceflica, medial al tendn del msculo


palmar mayor debe ser la causante del sangrado.
La vena baslica es la estructura vascular ms
frecuentemente seccionada ante un paciente
con esta sintomatologa.

Histoplasma capsulatum.
Mycobacterium tuberculosis (Bacilo de Koch).
Virus de la varicela - zoster.
Streptococcas pneumoniae (neumococo).
Pneumocystisjiroveci.

En las vacunas conjugadas frente a Haemophilus influenzae formadas por Polisacrido bac
teriano (PS) ms toxoide tetnico, la reaccin
inmunolgica en la que se basa el diseo y efica
cia de la vacuna es la siguiente:
1.

2.

3.

El linfocito B reconoce fragmentos del Polisacrido y presenta fragmentos de ste a las clulas T.
El linfocito B reconoce fragmentos del toxoide
tetnico y produce Anticuerpos frente al Polisacrido.
El linfocito B reconoce fragmentos del Polisacrido, presenta fragmentos del toxoide a las
clulas T y produce Anticuerpos frente al
toxoide.

Examen MIR. Versin 0. Enero 2008- PreguntasMIR

4.

5.

El linfocito B reconoce fragmentos del Polisacrido, presenta fragmentos del toxoide de los
linfocitos T y produce Anticuerpos frente al
Polisacrido.
El linfocito B reconoce fragmentos del toxoide, presenta fragmentos del toxoide a los linfocitos T y produce Anticuerpos frente al Polisacrido.

1.
2.
3.
4.
5.
251.

245.

Paciente de 23 aos sin patologa previa que


comienza con neumonas de repeticin, trombopenia e hipogammaglobulinemia. Qu diagns~
tico nos sugiere?:
1.
2.
3.
4.
5.

246.

248.

Sndrome de Peutz-Jeghers.
Sndrome de Cowden.
Sndrome de Gardner.
Sndrome de Lynch.
Poliposis Juvenil.

Carboxipeptidasa.
Tripsina.
Elastasa.
Lecitina.
Quimiotripsina.

La transformacin de testosterona a dihidrotestosterona lo cataliza la siguiente enzima:


1.
2.
3.
4.
5.

250.

3.

4.
5.

252.

2.
3.

CYP19 Aromatasa.
5-a Reductasa.
17,20Liasa.
17-aHidroxilasa.
20-Deshidrogenasa.

Lo abajo mencionado respecto a la Resistencia a


la insulina es cierto, EXCEPTO:

4.
5.
253.

La presencia de marcadores tumorales elevados, an usados como mtodo de screening,


tiene gran sensibilidad y especificidad
diagnstica.
La endocarditis marntica es una situacin
potencialmente embolgena, asociada a estados
de bipercoagulabilidad en procesos neoplsicos.
La elevacin de la enzima fosfatasa alcalina,
en este caso, refleja la presencia de metstasis
sea.
La elevacin de Biiirrubina directa apunta a
estados hemolticos.
El cuadro, en conjunto es diagnstico de
Hepatocarcinoma.

En la definicin de hipertensin arterial


resistente, se consideran las siguientes causas,
EXCEPTO:
1.

Los remanentes de quilomicrn son retirados de


la circulacin por el hgado mediante un proce
so que requiere ta siguiente apoprotena:
1. ApoB-100.
2. Apo E.
3. Apo C-n.
4. ApoA-I.
f 5. Apo (a).

249.

2.

Cul de las siguientes sustancias forma parte


de la secrecin biliar?:
1.
2.
3.
4.
5.

Una mujer de 46 aos acude a urgencias por


paresia de miembros derechos de instauracin
brusca. Sufri un cuadro de astenia y anorexia
desde hacia 2 meses. La paciente estaba ictrica.
Su tasa hemtica de Biiirrubina era 6 mg/dL (Bi
directa: 5 mg/dL). La fosfatasa alcalina 406 u/L
(n<127); El Ag. carcinoembrionario (CEA), 18,5
ng/mL (n<5); CA 19,9: 980 u/mL (n<37); Alfafetroproteina: 2 ng/mL (n<15). Seale la res
puesta correcta:
1.

Cul de los siguientes sndromes se caracteriza


por mutaciones en los genes de reparacin de
ADN, que se traduce en inestabilidad de microsatlites?:
1.
2.
3.
4.
5.

247.

Agarnmaglobulinemia ligada al cromosoma X.


Enfermedad granulomatosa crnica.
Sndrome variable comn de nmunodeficiencia.
Infeccin por VIH.
Sndrome de Hiper-IgM.

Su incidencia contina decreciendo en pases


occidentales.
Esta relacionada con la obesidad.
Se relaciona con niveles plasmticos bajos de
adiponectina.
Hay hiperinsulinismo e hiperglucemia.
Se relaciona con la falta de ejercicio muscular.

Cifras de Tensin Arterial Sistlica por encima de 180 rnrnHg.


Apnea Obstructiva del sueo.
Lesin orgnica irreversible o difcilmente
reversible.
Cumplimiento deficiente del plan teraputico.
Causa secundaria no sospechada.

Usted ve en consulta por primera vez a un pa


ciente de 45 aos diagnosticado de enfermedad
pulmonar obstructiva crnica (EPOC). Desde ios
25 aos haba fumado 5 cigarrillos al da (5
paquetes/ao). El sntoma fundamental es una
disnea de mnimos esfuerzos. La auscultacin
pulmonar es normal, aunque en la exploracin
fsica destaca la presencia de aumento de pre
sin venosa y edemas en ambos miembros infe
riores. Cul de las siguientes actuaciones con
sidera errnea en este paciente?:
1.

2.

Realizar una espirometra con prueba broncodilatadora para demostrar la existencia de obstruccin crnica al flujo areo.
Iniciar inmediatamente tratamiento con altas
Examen MIR. Versin 0. Enero 2008- PreguntasMIR

3.

4.
5.

254.

3.

4.

5.
259.

Giardiasis.
Salmonellosis.
Infeccin por Oostridium Difficile.
Adenoma velloso.
Adenocarcinoma de colon.

3.
4.
5.

El riesgo de tener un hijo afecto es 1/140.


Ninguna. Todos sus hijos sern sanos respecto
a la citada enfermedad.
Un 50% sern portadores, un 25% afectos y un
25% sanos no portadores.
Todos sus hijos sern afectos.
E riesgo de tener un hijo afecto es 1/70.

Un paciente de 60 aos es diagnosticado de


carcinoma epidermoide del lbulo inferior del
pulmn derecho (T2 NI M0). Tiene un FEVI
del 80% (superior a 2,5 L). Cul sera la mejor
opcin teraputica?:
1. Quimioterapia neoadyuvante y ciruga posterior.

Iniciar la respiracin boca a boca.


Cateterizar una va venosa perifrica y administrar 1 mg de adrenalina.
Asegurar que no existen cuerpos extraos en la
boca, levantar la mandbula e hperextender la
cabeza.
Hacer compresiones torcicas a razn de 80100 por minuto tras colocar al paciente sobre
plano duro.
Aplicar una desfibrilacin elctrica 200 joules.

Una mujer de 30 aos con 8 semanas de ameno


rrea acude a urgencias por presentar metrorragias y dolor hipogstrico desde hace unas horas.
La tensin arterial es de 120/80 mra Hg. El test
de embarazo es positivo. Por ecografa transvaginal se observa un endomtrio de 14 mm de
espesor sin imagen de saco gestacional dentro
del tero. Tampoco se observa imagen de saco
gestacional extrauterino. El hemograma es
normal y la determinacin de gonodatropina
corinica en plasma es de 3.000 mUl/ml. Cul
es la conducta ms aconsejable?:
1.

2.
3.
4.
5.
260.

Radioterapia con intencin curativa.


Ciruga y radioterapia posterior.
Ciruga slo.
Quimioterapia slo.

Durante (a celebracin de un concierto multitu


dinario de rock, un asistente al mismo sufre un
colapso sbito y se solicita su participacin
profesional entre tanto no llega el Sistema de
Emergencias Mdicas ya contactado. Su prime
ra valoracin del paciente es que est incons
ciente, con movimientos respiratorios de lucha,
ciantico y con pulso carotdeo presente. En
esta situacin. Cul debe ser su prioridad te
raputica?:
1.
2.

Mutaciones del gen X25 situado en la regin


cromosmea 9ql3 son causa de ataxia de Fredreich, de herencia recesiva. Un paciente afecto
de 30 aos acude a la consulta de consejo gen
tico con su pareja, sana y sin conocido paren
tesco. La frecuencia de portadores sanos en
Espaa se estima en uno entre setenta (1/70).
Qu probabilidad presenta la pareja de trans
mitir la enfermedad?:
1.
2.

257.

258.

Creatmina plasmtica.
Urea plasmtica.
Sodio urinario.
Eosinofilia en plasma.
Nivel de C3 en suero.

Paciente de 70 aos de edad sin antecedentes de


inters que ingresa en el hospital por una Neu
mona Neumoccica. Es tratado con Ceftriaxona evolucionando satisfactoriamente. A los 7
das del ingreso comienza con fiebre, diarrea
lquida maloliente, dolor clico abdominal y
leucocitosis. Cul, de los siguientes, le parece el
diagnstico ms probable?:
1.
2.
3.
4.
5.

256.

2.
3.
4.
5.

Cul de los siguientes parmetros resulta ms


til en el diagnstico diferencial de la necrosis
tubular aguda por frmacos?:
1.
2.
3.
4.
5.

255.

dosis de broncodilatadores, cortcoides inhalados y teoflna para mejorar su situacin clnica.


Cuestionar el diagnstico de EPOC y valorar
otras patologas que cursen con insuficiencia
cardiaca derecha.
Ampliar el estudio del paciente con radiografa
de trax y ecocardiograma.
Valorar la existencia de patologa vascular
pulmonar si se descarta alteracin parenquimatosa pulmonar y enfermedad cardiolgica.

Repetir la determinacin de gonodatropina


corinica en plasma y la ecografa cada 2-3 das y si la hormona aumenta adecuadamente a
lo esperado en un embarazo y sigue sin verse
saco gestacional intrauterino, indicar una laparoscopia.
Laparoscopa.
Legrado uterino.
Laparotoma.
Tratamiento con metrotrexato.

Cul de los siguientes frmacos debe ser utili


zado con gran precaucin por el riesgo derivado
de su capacidad para inducir la actividad de
varias de las isoenzimas CYP450 provocando
interacciones peligrosas?:
1.
2.
3.
4.
5.

Cloramfenicol.
Eritromicina.
Rifampicina.
Rjtonavir.
Itraconazol.

Examen MIR. Versin 0. Enero 2008- PreguntasMIR

También podría gustarte